+ All Categories
Home > Documents > FULL-LENGTH TEST 11R - Medical Mastermind...

FULL-LENGTH TEST 11R - Medical Mastermind...

Date post: 27-Jul-2018
Category:
Upload: lelien
View: 239 times
Download: 2 times
Share this document with a friend
76
FULL-LENGTH TEST 11R DO NOT OPEN THE TEST BOOK UNTIL YOU ARE TOLD TO DO SO. DIRECTIONS The FULL-LENGTH TEST 11R consists of the following four sections: Section 1: Physical Sciences 100 minutes 10 minute break Section 2: Verbal Reasoning 85 minutes 1 hour break Section 3: Writing Sample 60 minutes 10 minute break Section 4: Biological Sciences 100 minutes During the time given for a particular section, you may work only on that section. If you finish a section early, you may check your work within that section, but DO NOT GO BACK TO A PREVIOUS SECTION OR AHEAD TO A FORTHCOMING SECTION. Separate directions are provided within each section for each question type. Make sure you understand the directions before answering the questions. FILLING IN THE ANSWER GRID 1. Place all your answers on the separate Kaplan MCAT ® green answer grid provided. Using a #2 PENCIL ONLY, blacken the space corresponding to the letter of the answer choice you have selected. There should be only one answer per question. Do not be concerned if there are more spaces available on the answer grid than there are questions in a section. 2. Be sure your answer mark is dark and fills the space completely. Also, be sure all erasures are complete. The computer may misinterpret an incomplete erasure, and you will lose credit for that question. 3. Use your test booklet for any scratch work. DO NOT MAKE ANY STRAY MARKS ON THE ANSWER GRID. Erase any such marks. 4. The Answer Document for Section 3 is provided in a separate booklet. Use ink to complete this document. DO NOT USE INK ON ANY OTHER SECTION, OR ON THE COMPUTER-SCORED ANSWER GRID. 5. Your score on the MCAT ® is based on the number of correct answers. There is no penalty for incorrect answers or for answers left blank. Therefore, it may be to your advantage to answer every question. *MCAT is a registered trademark of the Association of American Medical Colleges Name: Date: MCAT 3264 SCAN CODE *
Transcript

FULL-LENGTH TEST 11RDO NOT OPEN THE TEST BOOK UNTIL YOU ARE TOLD TO DO SO.

DIRECTIONS

The FULL-LENGTH TEST 11R consists of the following four sections:

Section 1: Physical Sciences 100 minutes10 minute break

Section 2: Verbal Reasoning 85 minutes1 hour break

Section 3: Writing Sample 60 minutes10 minute break

Section 4: Biological Sciences 100 minutes

During the time given for a particular section, you may work only on that section. If you finish a section early, youmay check your work within that section, but DO NOT GO BACK TO A PREVIOUS SECTION OR AHEAD TOA FORTHCOMING SECTION.

Separate directions are provided within each section for each question type. Make sure you understand thedirections before answering the questions.

FILLING IN THE ANSWER GRID

1. Place all your answers on the separate Kaplan MCAT® green answer grid provided. Using a #2 PENCIL ONLY,blacken the space corresponding to the letter of the answer choice you have selected. There should be only oneanswer per question. Do not be concerned if there are more spaces available on the answer grid than there arequestions in a section.

2. Be sure your answer mark is dark and fills the space completely. Also, be sure all erasures are complete. Thecomputer may misinterpret an incomplete erasure, and you will lose credit for that question.

3. Use your test booklet for any scratch work. DO NOT MAKE ANY STRAY MARKS ON THE ANSWER GRID.Erase any such marks.

4. The Answer Document for Section 3 is provided in a separate booklet. Use ink to complete this document. DONOT USE INK ON ANY OTHER SECTION, OR ON THE COMPUTER-SCORED ANSWER GRID.

5. Your score on the MCAT® is based on the number of correct answers. There is no penalty for incorrect answersor for answers left blank. Therefore, it may be to your advantage to answer every question.

*MCAT is a registered trademark of the Association of American Medical Colleges

Name: Date:

MCAT3264

SCAN CODE*

Full Length Test 11R 11/21/03 6:23 PM Page 1

© 2004 Kaplan, Inc.All rights reserved. No part of this book may be reproduced in any form, by photostat,

microfilm, xerography, or any other means, or incorporated into any information retrieval system,electronic or mechanical, without the written permission of Kaplan, Inc.

This book may not be duplicated or resold, pursuant to the terms of your KaplanEnrollment Agreement.

PHYSICAL SCIENCES

DIRECTIONS: Most of the questions in the PhysicalSciences test are organized into groups, with adescriptive passage preceding each group ofquestions. Study the passage, then select the singlebest answer to each question in the group. Some of thequestions are not based on a descriptive passage; youmust also select the best answer to these questions. Ifyou are unsure of the best answer, eliminate thechoices that you know are incorrect, then select ananswer from the choices that remain. Indicate yourselection by blackening the corresponding circle onyour answer sheet. A periodic table is provided belowfor your use with the questions.

1

H

1.0

2

He

4.0

3

Li

6.9

4

Be

9.0

5

B

10.8

6

C

12.0

7

N

14.0

8

O

16.0

9

F

19.0

10

Ne

20.2

11

Na

23.0

12

Mg

24.3

13

Al

27.0

14

Si

28.1

15

P

31.0

16

S

32.1

17

Cl

35.5

18

Ar

39.9

19

K

39.1

20

Ca

40.1

21

Sc

45.0

22

Ti

47.9

23

V

50.9

24

Cr

52.0

25

Mn

54.9

26

Fe

55.8

27

Co

58.9

28

Ni

58.7

29

Cu

63.5

30

Zn

65.4

31

Ga

69.7

32

Ge

72.6

33

As

74.9

34

Se

79.0

35

Br

79.9

36

Kr

83.8

37

Rb

85.5

38

Sr

87.6

39

Y

88.9

40

Zr

91.2

41

Nb

92.9

42

Mo

95.9

43

Tc

(98)

44

Ru

101.1

45

Rh

102.9

46

Pd

106.4

47

Ag

107.9

48

Cd

112.4

49

In

114.8

50

Sn

118.7

51

Sb

121.8

52

Te

127.6

53

I

126.9

54

Xe

131.3

55

Cs

132.9

56

Ba

137.3

57

La *

138.9

72

Hf

178.5

73

Ta

180.9

74

W

183.9

75

Re

186.2

76

Os

190.2

77

Ir

192.2

78

Pt

195.1

79

Au

197.0

80

Hg

200.6

81

Tl

204.4

82

Pb

207.2

83

Bi

209.0

84

Po

(209)

85

At

(210)

86

Rn

(222)

87

Fr

(223)

88

Ra

226.0

89

Ac †

227.0

104

Rf

(261)

105

Ha

(262)

106

Unh

(263)

107

Uns

(262)

108

Uno

(265)

109

Une

(267)

*

58

Ce

140.1

59

Pr

140.9

60

Nd

144.2

61

Pm

(145)

62

Sm

150.4

63

Eu

152.0

64

Gd

157.3

65

Tb

158.9

66

Dy

162.5

67

Ho

164.9

68

Er

167.3

69

Tm

168.9

70

Yb

173.0

71

Lu

175.0

90

Th

232.0

91

Pa

(231)

92

U

238.0

93

Np

(237)

94

Pu

(244)

95

Am

(243)

96

Cm

(247)

97

Bk

(247)

98

Cf

(251)

99

Es

(252)

100

Fm

(257)

101

Md

(258)

102

No

(259)

103

Lr

(260)

PERIODIC TABLE OF THE ELEMENTS

Full Length Test 10R 11/21/03 7:00 PM Page 3

Physical SciencesTime: 100 Minutes

Questions 1–77

DO NOT BEGIN THIS SECTION UNTIL YOU ARE TOLD TO DO SO.

MCAT FL Test 11R 11/21/03 7:36 PM Page 1

Passage I (Questions 1–7)

The Diamond has long been valued for its brilliance,fire, and utility, but the most fascinating aspect of the gemare the chemical qualities that allow for it to display suchremarkable beauty. Diamond is the least compressible andstiffest substance. It exhibits very little thermal expansion,yet has remarkable thermal conduction properties—a fea-ture which lends to diamonds being called “ice.”

The diamond is essentially a dense crystalline lattice ofcarbon bound by covalent bonds. The lattice is so wellorganized in both diamond and graphite that standardentropy values are near zero.

Substance Sº (J/K · mole)

C(diamond) 2.44

C(graphite) 5.69

The conversion of graphite to diamond is an intenseprocess that involves the interchange of trigonal planarbonds to a tetrahedral conformation. The process is excep-tionally slow and involves unstable intermediates that havea high degree of angle strain. The conversion of graphite todiamond is favored at high temperatures and pressuressuch as those found about 100 kilometers below the sur-face of the earth. The opposite is true at low temperatureand pressure.

Finally, a diamond’s incredibly high index of refractionallows for a properly cut diamond to completely reflectany light shined directly on it. The ability to reflect almost100% of the light directed into it gives a diamond its char-acteristic brilliance.

Several graduate students set up an experiment todirectly measure the dispersion constant of diamonds. Thedispersion constant is defined as:

� = (N1 – 1) / ∆N

where ∆N is the difference between the indices of refrac-tion for the shortest light and the longest light (usually vio-let and red) and N1 is the index of refraction for light witha length in-between the two extremes.

Experiment 1

A Helium lamp was placed in front of a collimator, which

was used to focus the light beam into vertical slits. The

light was aimed at a precisely cut prism made from

synthetic diamond. 9 colored lines that make up the

expected helium spectrum were identified and the indices

of refraction were calculated for 3 of them yielding the

data listed in Table 1.

Light Color Index

Red 2.410

Yellow 2.418

Violet 2.454

Experiment 2

The same experiment was performed only this time a

sodium lamp was used yielding the indices listed in table

2. (no yellow light was observed)

Light Color Index

Red 2.413

Violet 2.452

1. What is the speed of red light inside a diamond?

A. c = 1.25 × 108 m/s

B. c = 3.00 × 108 m/s

D. c = 7.25 × 108 m/s

E. c = 2.75 × 108 m/s

2. If a 10g diamond were located 1000km below thesurface of the earth, what is the magnitude of thegravitational force acting on it?

A. 9.8 N

B. 0.098 N

C. 0.14 N

D. 0.073 N

2

GO ON TO THE NEXT PAGE.

MCAT FL Test 11R 11/21/03 7:36 PM Page 2

3. Diamond is one of the hardest substances known toman, while carbon is one of the softest, yet both areaggregates of carbon atoms. Which of the followingbest explains this?

A. Graphite has greater angle strain than diamond.

B. Both consist of sp3-hybridized orbitals, but onlydiamond has tetrahedral molecular geometry.

C. The carbon atoms of graphite each have anunhybridized 2p orbital.

D. Diatomic carbon normally contains 3 pi bonds;this stable state is mimicked by diamond.

4. Why were the values of n for violet light different inthe two experiments?

A. The violet observed from a sodium lamp has aslightly different wavelength than the violetfrom a helium lamp.

B. A sodium lamp emits light with higher energythan a helium lamp; therefore its violet light willtravel faster than the corresponding visible lightfrom helium lamps in a given medium.

C. Angles of refraction that small cannot be deter-mined without introducing large amounts oferror. The actual values should have been thesame.

D. Photons leaving sodium atoms give off a portionof their kinetic energy to “escape” the largernucleus. This loss can be detected as a change inrefractive index.

5. Gallium phosphide has the one of the highest knownoptical densities with an index of refraction of 3.02for red light. Which of the following would thereforebe true?

A. The dispersion constant would be lower for gal-lium phosphide than for diamond.

B. Total internal reflection cannot occur for lighttraveling from gallium phosphide into water (forwater n = 1.33).

C. Gallium phosphide will have a higher densitythan mercury at room temperature.

D. 9 colored lines would be detected exiting aprism made of gallium phosphide after illumi-nation by a helium lamp.

6. Which of the following would exhibit total internalreflection at the smallest angle of incidence movingfrom diamond to air if air has a divergence constantof 0?

A. Red Light

B. Ultraviolet Light

C. Orange Light

D. Violet Light

7. Young’s modulus is important for many industrialapplications. Which among these materials has ahigher young’s modulus?

A. Fullerene

B. Graphite

C. Diamond

D. All carbon composite forms have the sameyoung’s modulus

3

GO ON TO THE NEXT PAGE.

MCAT FL Test 11R 11/21/03 7:36 PM Page 3

Passage II (Questions 8–13)

Electroplating allows for an efficient cost-effectivemeans of applying a thin veneer of an expensive substancesuch as gold onto a metallic item. This can be done for aes-thetic purposes, such as in jewelry. It can also be used tocoat items for protective purposes, such as corrosion pre-vention. A variety of materials can be used as a platingmaterial, and a variety of materials have properties whichallow them to be plated easily.

In a certain electroplating apparatus used to plate alu-minum eyeglass frames, a bar of gold or copper is placedin an acidic solution and attached via a battery to the itemto be plated. The eyeglasses to be plated are also placed inthe solution. A voltage is applied to create an electrolyticcell, which plates the material onto the eyeglass frames.

Au3++ 3e → Au 1.50 V

Fe3+ + 1e → Fe2+ 0.70 V

Cu2+ + 2e → Cu 0.34 V

Zn2+ + 2e → Zn 0.76 V

Cr2+ + 2e → Cr 0.91 V

Al3+ + 3e → Al 1.66 V

Ca2+ + 2e → Ca 2.87 V

Table 1–Standard reduction potentials

(Note: Faraday’s constant is 96,487 C / 1 mol e–)

8. The total amount of gold to be deposited onto a pairof glasses made of aluminum is 0.5 grams. How longshould the plating process be performed to depositthis amount of gold?

A. 73 seconds

B. 146 seconds

C. 24 seconds

D. 48 seconds

9. Which of the following statements is correct?

A. The gold bar is the anode, the aluminum is thecathode.

B. The gold is reduced, the aluminum is the oxi-dizing agent.

C. The aluminum is oxidized, the gold is reduced.

D. The aluminum is the cathode, the gold isreduced.

10. Suppose the glasses are to be plated with copperinstead of gold. Which of the following statements istrue?

A. The reaction will require a lower voltage andproceed at the same speed for an equal current.

B. The reaction will require a lower voltage butproceed more quickly for an equal current.

C. The reaction will not require an applied voltage.

D. The reaction will require the same voltage butproceed more slowly for an equal current.

11. A certain pair of eyeglasses is to be coated whichconsists of parts made from both Zinc and Alu-minum. Which of the following describes the resultwhen plating with Gold?

A. The Zinc and Aluminum will be plated equally.

B. The Aluminum will be plated more quickly thanthe Zinc will be plated.

C. The Zinc will not be plated.

D. The Aluminum will not be plated.

12. What voltage should be applied to ensure that Cop-per will plate onto a set of eyeglasses made from Alu-minum?

A. 1.32 Volts

B. 3.16 Volts

C. 0.16 Volts

D. 2.00 Volts

13. Consider a highly acidic solution in an iron jar usedfor electroplating. The plating substance is Copperand the item to be plated is Aluminum. When thevoltage is applied what best describes a possibleresult?

A. Iron will be plated onto the Copper bar.

B. Copper will be plated onto the sides of the con-tainer.

C. Iron will be plated onto the Aluminum.

D. Aluminum will be plated onto the Copper.

4

GO ON TO THE NEXT PAGE.

MCAT FL Test 11R 11/21/03 7:36 PM Page 4

Questions 14 through 18 are NOT based on a descriptive passage.

14. A rifle bullet is fired horizontally off the top of anocean based drill site. If air resistance is negligibleand the vertical distance to the water below is 200feet, what additional information is needed to deter-mine the time required for the bullet to hit the water?(g = 32 ft/sec2)

A. Horizontal distance the bullet travels

B. Initial velocity of the bullet

C. Mass of the bullet

D. No additional information

15. Which of the following choices lists elements, fromleft to right, with decreasing atomic radii?

A. Li, Be, B, C, N

B. Be, Mg, Ca, Sr, Ba

C. Li, Mg, Sc, Zn, Ta

D. Cu, Ag, Au

16. All of the following statements concerning the pho-toelectric effect are true EXCEPT

A. The kinetic energy of the emitted electrons isequal to the energy of the light waves.

B. As the intensity of the light increases, the num-ber of electrons emitted per unit time shouldincrease.

C. If the frequency of the light wave increases andthe intensity remains constant, the kineticenergy of the emitted electrons increases.

D. If light of extremely low frequency is incidentupon a photosensitive metallic surface, an elec-tron will not be emitted regardless of how longthe light is incident upon that surface.

17. All of the following are false about the elastic limit ofa solid EXCEPT:

A. It also applied to liquids and gases.

B. If a solid is deformed beyond the elastic limit, itwill not return to its original shape when theapplied force is removed.

C. Hooke’s laws correspond to forces exceedingthe elastic limit.

D. When a force is applied that exceeds the elasticlimit of a solid, the amount that the solid com-presses or expands is linearly proportional to theapplied force.

18. An ideal gas is contained in a one-liter vessel at 350K. What additional information is sufficient to deter-mine the pressure of the gas?

A. The molecular weight of the gas

B. The number of gas molecules

C. The molecular weight and the mass of the gas

D. Two of the above

5

GO ON TO THE NEXT PAGE.

MCAT FL Test 11R 11/21/03 7:36 PM Page 5

Passage III (Questions 19–26)

Because of their energy efficiency in reactions, cata-lysts are widely employed in chemical synthesis. Primitivecatalytic systems such as silica-alumina gels are formed byreplacing some of the silicon atoms of a silica (silicon-oxygen) gel with aluminum atoms. Because each trivalentaluminum atom is electron deficient relative to silicon, anacidic site occurs wherever an aluminum atom replaces sil-icon.

Solid acid catalysts have proved particularly useful inorganic synthesis, especially in hydrocarbon crackingreactions in which larger, less mobile molecules are brokendown into smaller, more mobile products. Such a reactionmay be expressed by a generalized reaction equationbetween an “oil” and its products. For example,

n C12H18 n C2H4 + n C6H6 + n C4H8(“oil”)

As a result of the acidic character of the catalyst sur-face, cracking of hydrocarbons takes place via heterolyticor ionic cleavage of a bond on a carbocation center.

The main drawback to these catalysts is that their irreg-ular pore size also permits formation of larger, unwantedproducts that quickly render the catalysts useless. Usingzeolite catalysts, which are also silica-alumina in nature,but with highly regular geometric shapes that possesspores of small, fixed size, has greatly alleviated this prob-lem. Catalytic activity occurs mostly inside the cavities ofthe catalyst; access to these sites is controlled by the poresize. As a result, these catalysts possess a significantadvantage in synthesis reactions over the primitive silica-alumina catalysts. For example, the gas phase acid-cat-alyzed conversion of methanol and ammonia to methylatedamines in a silica-alumina gel produces a mono-/di-/trimethylamine mixture at equilibrium in the mole ratio15:23:62 (a mass ratio of 9:20:71) predicted by thermody-namics. Using a strongly acidic Rho zeolite catalyst, how-ever, gives a mole ratio of 14:82:04 for the product mixtureapparently because most of the tri-substituted product istrapped in the pores of the catalyst.

One of the interesting characteristics of zeolite cata-lysts is that many of their physico-chemical properties canbe predicted on the basis of their Si/Al ratio. The follow-ing table lists a few of these properties. Probably the mostintriguing property of acid catalysts is that the fewer thenumber of acid groups, the stronger these acid groups are.

Si/Al Atomic Ratio Zeolites Properties

- Low stability of frame-work, with generally largerpores

- Low stability in acid

Low (1 – 1.5) A - High stability in base

X - High concentration of acidgroups with moderate acidstrength

Intermediate Eriunite __________(2 – 5) Y

- High stability of frame-work, with generallysmaller pores

- High stability in acid

High (10 - ∞) ZSM-5 - Low stability in base

- Low concentration of acidgroups with high acidstrength

19. Based on the terminology of the passage, which gen-eralized reaction can NOT occur in a catalyzedcracking process?

A. m C11H16 m C3H6 + m C6H6 + m C2H4

B. n C12H20 n C2H4 + n C4H8 + n C6H6

C. p C13H20 p C3H6 + p C4H8 + p C6H6

D. q C9H18 q C2H4 + q C3H6 + q C4H8

6

GO ON TO THE NEXT PAGE.

catalyst/heat

catalyst/heat

catalyst/heat

catalyst/heat

catalyst/heat

MCAT FL Test 11R 11/21/03 7:36 PM Page 6

20. Which of the labeled sites in the figure below is themost likely to be an acidic site when used as a cata-lyst for cracking hydrocarbons?

A. A

B. B

C. C

D. D

21. If the partial pressures of methanol and water vapor areadjusted until they are equal, what is the expressionfor ∆G° for the given reaction using a silica-aluminagel solid acid catalyst?

CH3NH2(g) + CH3OH(g) (CH3)2NH(g) + H2O(g)

A. –RT ln (23/15)

B. –RT ln (82/14)

C. –RT ln (20/9)

D. –RT ln (15/23)

22. In the reaction to form methylated amines frommethanol and ammonia, which is the most likely rea-son that using a zeolite catalyst produces more sec-ondary amine than tertiary amine?

A. Only tertiary amines can form inside the poresof the catalyst.

B. Tertiary amines are too large to form inside thepores of the catalyst.

C. The tertiary product undergoes a reverse reac-tion to re-form the secondary product.

D. Secondary amines can no longer undergo thedisproportionation reaction that forms tertiaryamines and at the same time re-forms primaryamines.

23. Which of the following synthetic systems would bethe least likely to use a ZSM-5 zeolite catalyst?

A. The synthesis of ethylbenzene from benzeneand ethene in acidic medium.

B. The synthesis of gasoline (isooctane) frommethanol in basic medium.

C. The synthesis of ethylamine from ethanol andammonia in acidic medium.

D. The synthesis of p-xylene (1,4-dimethylben-zene) and benzene from the disproportionationof toluene (methylbenzene) in neutral medium.

24. Based upon the passage, which of the followingstatements about the acid-catalyzed conversion ofmethanol and ammonia to the mono-, di- andtrimethylated amines is (are) true?

I. The acidic catalyst used has a relatively high sili-con to aluminum ratio.

II. If the conversion were uncatalyzed, the mono-,di- and trimethylated amine product mole ratiowould be 15:23:62.

III. The ∆G° for the catalyzed reaction is the same asthat for the uncatalyzed reaction.

A. II and III only

B. I, II, and III

C. III only

D. I and II only

25. Which of the following equations CANNOT repre-sent a possible solid acid-catalyzed hydrocarboncracking reaction?

A. C12H20 → C6H14 + C6H6B. C15H32 → 5 CH4 + C4H6 + C6H6C. C20H26 → 2 CH4 + 3 C6H6D. C11H18 → CH4 + 2 C2H4 + C6H6

O Si O Al O Si O Al O Si O

O Al O Al O Si O Si O Si O

O O O O O

O

A B C D

7

GO ON TO THE NEXT PAGE.

MCAT FL Test 11R 11/21/03 7:36 PM Page 7

26. A ZSM-5 catalyst will LEAST successfully crackwhich of the following hydrocarbon “oils” at lowtemperatures?

A.

B.

C.

D.

CH3

CH3

CH3

CH3

CH3

CH3

CH3CH3

8

GO ON TO THE NEXT PAGE.

MCAT FL Test 11R 11/21/03 7:36 PM Page 8

Passage IV (Questions 27–34)

A depiction of shielding against the radiative particlesproduced in a nuclear fission reactor is shown in the figurebelow. Here N particles impinge upon a slab of unit surfacearea and unit thickness ∆L, and ∆N is the numberabsorbed.

The mechanism of particle absorption may be modeledas a first order process in which the number of radiativeparticles absorbed per unit thickness is directly propor-tional to the number of particles entering the slab. This isrepresented by the equation

= kN

Equation 1

(The negative sign indicates an absorption or removalof particles.) The efficiency of the shielding is related tothe quantity –∆N/N, the fraction of particles absorbed bythe material.

Neutrons and � rays are the most penetrating radiativeparticles and are therefore the most dangerous; shieldingagainst these particles stops all other decay particle types.All gamma particles are stopped by a combination of threemechanisms. Low energy � rays are absorbed by the Kshell (lowest energy) electrons of atoms in the shielding;once freed, the electrons are then absorbed by the shield-ing. Moderate energy � rays undergo Compton scatteringwith electrons. At high energies, the coulomb field of anucleus in the shielding can convert the photon into ae+–e– pair; the cross-section � (approximately equivalentto the probability of occurrence) for this conversion reac-tion is given by

� � kZ2 (E – 1.02 MeV)

Equation 2

where Z is the atomic number of the nucleus and the �ray energy E is greater than 1.02 MeV. High-energy neu-trons entering the shielding are slowed by collisions untilthey can be captured by atoms such as 6Li and 10B. Excessenergy appears as heat and � rays (produced outside thereactor core, these are called secondary � rays). Lowenergy neutrons (< 10 keV) undergo elastic collisions,with very small capture cross-section values.

An interesting comparison may be extracted from thefirst order model for reactor shielding. Rearrangement andsuitable mathematical processes transform Equation 1 into

N = Noe–kl

Equation 3

which is of the same form as the equation for radioactivenuclear decay. This means that the mathematical propertiesof the absorption process are similar to those of nucleardecay.

27. The fraction of particles absorbed by the shielding

A. increases with increasing thickness of theshielding.

B. decreases with increasing density of the shield-ing material.

C. is independent of the shielding thickness.

D. is always greatest for lead than for any othermaterial.

28. If a 2.0 cm thick shielding absorbs one half of the �particles that strike it, what is the approximate per-centage of particles that are absorbed by a 7.0 cmthick shielding?

A. 14%

B. 63%

C. 84%

D. 91%

29. A manufacturer of a nuclear reactor whose decayprocess involves high energy � ray production, wish-ing to provide the most efficient shielding, will electto use

A. 207Pb, because although it will not interact with� rays, it is denser than 10B.

B. 184W, because nonmetals are better � rayabsorbers than metals.

C. 207Pb, because it has more protons than 184W.

D. 184W, because it has more neutrons than 207Pb.

∆N�∆D

N particles N - ∆N particles∆L

9

GO ON TO THE NEXT PAGE.

MCAT FL Test 11R 11/21/03 7:36 PM Page 9

30. The symbol “�?” is often replaced by the expression“1/� ?”. What are the units of �

A. m

B. s

C. m–1

D. kg·m–3

31. All of the following statements are true EXCEPT:

A. As � ray wavelength decreases, interaction withK shell electrons decreases.

B. � particles are readily absorbed by the shield-ing.

C. All � rays are absorbed, though neutron absorp-tion is not necessarily complete.

D. The capture cross-section for a neutronincreases with increasing energy.

32. Which of the following statements is (are) correct?

I. The mechanism for removing low energy � raysis the photoelectric effect.

II. Nuclear decay is the only source of � rays in anuclear fission reactor.

III. The Compton scattering of electrons by moderateenergy � rays involves elastic collisions.

A. I only

B. II only

C. II and III only

D. I and III only

33. Which of the following nuclear reactions can occurin the reactor shielding?

A. Li + n → Be + e+

B. B + � → B + 2 n

C. Li + n → B + 2e– + 2e+

D. B + n → Li + He

34. At neutron energies of 1 – 10 keV, neutrons undergoelastic scattering within the shielding. What is theapproximate amount of heat dissipated by stopping aneutron that enters the shielding with a kinetic energyof 6.25 MeV? (1 eV = 1.6 × 10–19 J)

A. 1.0 × 10–18 kJ

B. 1.0 × 10–12 kJ

C. 1.0 × 10–15 kJ

D. 1.6 × 10–16 kJ

42

73

10

105

85

10

63

10

95

105

74

10

63

10

GO ON TO THE NEXT PAGE.

MCAT FL Test 11R 11/21/03 7:36 PM Page 10

Passage V (Questions 35–40)

Nickel-Cadmium cells are used to make rechargeablebatteries used extensively in devices ranging from portablestereos to cordless power tools. A Ni-Cd battery is an elec-trochemical cell in which all of the redox components aresolids and no salt bridge is necessary, as one of thereagents can perform its function. The Cadmium serves asthe anode and the Nickel serves as the cathode. KOH isgenerally used as an electrolyte.

Cd(OH)2 + 2e– → Cd + 2OH– –0.83 V

NiO(OH) + H2O + e– → Ni(OH)2 + OH– +0.52 V

Table 1–Reduction potentials

The advantages of the Ni-Cd cells over standard lead-acid batteries include the high cycles of use, often several1000’s of recharge cycles, and long shelf life, up to 50%longer than a typical Nickel metal hydride battery. The dis-advantages include the toxicity of Cadmium, a heavymetal, a higher cost of production, lower power densitiesand a memory effect with respect to charging. This mem-ory effect is caused by the changes in the negative, or cad-mium plate, due to the formation of cadmium crystals.

The symptom of this memory effect is that if the bat-tery has not been fully discharged before recharging, lesspower will be available. Another effect is known as “lazybattery effect”. This is the result of repeated overchargingof a Ni-Cd battery and results in a rapid discharge over abrief period of operation. This effect can be eliminatedwith a few thorough discharge cycles.

(Note: Faraday’s constant is 96,487 C/mol e-)

35. Which shows the overall reaction in a Ni-Cd battery?

A. 2Ni(OH)2 + Cd(OH)2 → 2NiOOH + Cd + 2H2O

B. 2NiOOH + 2Cd + 2H2O → Ni(OH)2 + Cd(OH)2C. 2NiOOH + Cd + 2H2O → 2Ni(OH)2 + Cd(OH)2D. Ni(OH)2 + Cd(OH)2 → NiOOH + Cd + H2O

36. What is the potential of a Ni-Cd battery that consistsof 3 cells in series?

A. 4.05 V

B. 1.35 V

C. 0.31 V

D. 0.93 V

37. A Ni-Cd cell with 20% of its maximum charge isconnected to a power source that supplies 0.1A ofcurrent, driving an electrolytic reaction. How manygrams of Ni are reacted in 1 minute?

A. 11.1 mg

B. 1.9 mg

C. 7.4 mg

D. 3.7 mg

38. Which of the following statements is true?

A. The reaction is spontaneous when Cadmium isoxidized.

B. The reaction is spontaneous when Nickel is oxi-dized.

C. The reaction is spontaneous when Nickel is thereducing agent.

D. The reaction is spontaneous when Cadmium isthe oxidizing agent.

39. What is the oxidation state of Nickel before the reac-tion takes place?

A. –3

B. +2

C. +3

D. –2

40. A Ni-Cd battery used in a calculator is left unat-tended for a long period of time will eventuallydevelop corrosion in the form of a white crystallinesubstance near the battery terminals. Given that thecorrosion is caused by the leaking of KOH, and thebattery is exposed to air, which is most likely the cor-rosive residue?

A. K+

B. KOH

C. K2CO3D. Cd

11

GO ON TO THE NEXT PAGE.

MCAT FL Test 11R 11/21/03 7:36 PM Page 11

Questions 41 through 45 are NOT based on a descriptive passage.

41. Two blocks whose masses are m1 and m2 start fromrest and slide down two planes of equal lengthinclined at angles θ1 and θ2 respectively. Given thatthe coefficient of kinetic friction is equal in the twocases, that m1 is greater than m2, and that θ1 isgreater than θ2, all of the following statements aretrue EXCEPT:

A. The time to slide down the plane depends onacceleration and the length of the plane.

B. Since both planes have the same length, the timeonly depends on this acceleration.

C. Regardless of their masses, m1 slides downfaster because of the larger angle of inclination.

D. The ratio of m to θ determines which blockslides down faster.

42. Two balls having the same mass are shot from thesame point with the same initial speed at the sametime, but at different angles to the horizontal. All ofthe following statements are true EXCEPT:

A. The ball launched at the larger angle hits theground last.

B. The ball launched at the smaller angle hits theground first.

C. The two balls will hit the ground at the sametime.

D. Mechanical energy is potential energy pluskinetic energy

43. If the ∆H of a reaction is found to be negative, whatcan we conclude about the reaction?

A. The reaction will proceed spontaneously.

B. The reaction will not proceed spontaneously.

C. Heat is absorbed during the reaction.

D. Heat is released during the reaction.

44. A mass m undergoes simple harmonic motion frommaximal displacement point A to maximal displace-ment point C with point B being the center restingposition. What can be said about the restoring forceacting on the mass, assuming that the mechanicalenergy of the system remains constant?

A. maximum at A and in the direction of accelera-tion

B. maximum at B and in the direction of accelera-tion

C. maximum at C and in opposite direction ofacceleration

D. minimum at C and in direction of acceleration

45. Which of the following mirror(s) can produce a realinverted image?

A. Convex only

B. Concave only

C. Plane only

D. All of the above

12

GO ON TO THE NEXT PAGE.

MCAT FL Test 11R 11/21/03 7:36 PM Page 12

Passage VI (Questions 46–49)

Capacitors are devices that can store electrical chargeand energy.

Engineers use them in many applications such as elec-tronic circuits and power supply regulators. Between theplates of a charged capacitor there is a voltage V due towhich an electric field E develops. For a capacitor withsmall distance d between its plates this electric field is

E = .

A charged capacitor stores electric energy, whichequals the work done by the agent that charged the capac-itor (battery or generator). The expression for the energy is

U0 = CV2 where C is the capacitance. This energy is

released back when the capacitor is discharged. One canconsider this energy stored in the electric field. The densityof the electric field energy for the vacuum is

u0 = = = ε0E2 where A is the area of the

plates and ε0 = 8.85 × 10–12 C2 / N • m2 is the permitivityof the vacuum.

If a dielectric is present, ε0 in the previous expressionmust be replaced by ε = Kε0, where K is the dielectric con-stant. Therefore the presence of dielectric influences theenergy density and the capability of energy storage of thecapacitor. If you insert a dielectric after the capacitor hasbeen charged and disconnected from the battery the energydensity drops. The reason is that since the charge remainsconstant and the capacitance increases by a factor of K, thevoltage drops by a factor of K and so does the energy andenergy density. So, in this situation the energy and energy

density are given by U = and u = correspondingly.

Each type of dielectric can withstand a maximum electricfield beyond which a spark breaks through it and thecapacitor is discharged and usually destroyed. Effectivelythe dielectric, which is an insulator, becomes a conductorwhen the electric field exceeds this maximum value. Thisphenomenon is called “dielectric breakdown”. Thus foreach dielectric there is a maximum possible energy den-sity. Further, the capacitance of a capacitor with plates ofgiven size cannot be increased arbitrarily by decreasing thedistance d since for a certain distance d the electric fieldwould increase beyond its allowed maximum and a sparkwould then develop. Dielectrics with high K are used toobtain high capacitance in combination with a capacitor ofsmall physical size.

46. A charged capacitor with mica (K = 7) as dielectric isinsulated and then immersed in water (K = 80).

A. Its capacitance will increase

B. Its capacitance will decrease

C. Its capacitance will remain the same

D. A spark will develop

47. A capacitor of given capacitance C, working at volt-age V cannot be made arbitrarily small because

A. the technology for small conducting plates isnot yet adequately developed.

B. a smaller size would necessarily decrease C.

C. a smaller size would decrease the energy storedin the capacitor.

D. a smaller size would result to a dielectric break-down.

48. A capacitor consisting of two flat, parallel plates sep-arated by vacuum is connected to a battery with volt-age V. Then its plates are pulled apart to a distancedouble the original one. By what factor does theenergy density between the plates change?

A. 2

B.

C. 4

D.

49. Two capacitors with equal capacitance C are con-nected in series. Their free ends are then connected toa battery with voltage V. The total energy stored inthe capacitors is

A. CV2

B. CV2

C. CV2

D. CV21�8

1�4

1�2

1�4

1�2

u0�KU0�K

1�2

CV2�AD

1�2

U0�Ad

1�2

V�d

13

GO ON TO THE NEXT PAGE.

MCAT FL Test 11R 11/21/03 7:36 PM Page 13

Passage VII (Questions 50–56)

The collision of baseball and bat is a complex eventinvolving forces of great magnitude and transfer of largeamounts of energy. As a major leaguer hits a fastball withhis wooden bat, the ball itself is compressed like a spring,flattening to nearly one-third its original diameter. Thekinetic energy of the pitch is transferred into spring poten-tial energy in the ball. This is, however, an inefficienttransfer; only 30 percent of the baseball’s kinetic energy isconserved. Other types of balls of greater compressibility,(lower spring coefficient, k) can store up to 45 percent ofthe kinetic energy of the pitch. The bat’s momentum cre-ates an impulse of bat-on-ball, transferring a huge amountof energy in under a millisecond. As the ball leaves the bat,often at a faster speed than it was pitched, essentially all ofits energy is kinetic.

When the ball hits the bat, some of the energy of thecollision is lost to the bats vibration. The bat itself res-onates in a very similar pattern to an open-ended pipe withthe second and third harmonics most prevalent. These twowaves constructively and destructively interfere. Wherethey destructively interfere the most, the bat bends theleast. Thus a collision which occurs on or near the nodeswill produce the least vibration and the greatest transfer ofenergy to the ball. This is what creates the “sweet spot” ona bat and what accounts for the characteristic “crack” of awell-hit ball. The sweet spot is the node of the second har-monic nearest the thicker end of the bat. The frequency ofboth the second harmonic and the “crack” sound is 170 Hz.However, researchers now believe that there is a “sweetzone” that extends from the old sweet spot to the node ofthe third harmonic which is closest to the thick end of thebat.

One way players have attempted to produce morepower is to cork their bats. In other words, they removesome of the hard wood from the thick part of the bat andadd cork. This reduces the mass of the bat and thus allowsthem to swing it faster, increasing the maximum velocityof the bat through the strike zone. A researcher performedan experiment to test the actual effect of a corked bat.Some of her data appear in Table 1.

Figure 1. Nodes of second harmonic

Figure 2. Nodes of third harmonic

Mass VMax

Uncorked bat 1.00 kg 20

Corked bat 0.91 kg 22

Standard baseball 0.145 kg

Table 1–Corked bat experience

50. How long is the “sweet zone” on a 81 cm woodenbat?

A. 6.75 cm

B. 13.5 cm

C. 20.25 cm

D. 3.375 cm

51. A batter hits a changeup traveling at V m/s parallel tothe horizon. If the ball experiences an average accel-eration of a m/s2 toward the pitcher, how far will theball travel before it hits the ground? Assume the ballcontacts the bat for tc seconds and stays in the air fortair seconds.

A. (–V + a tcir)tc meters

B. Vtair + � � ata2 meters

C. Vtair meters

D. tair (atc + V) meters

1�2

14

GO ON TO THE NEXT PAGE.

MCAT FL Test 11R 11/21/03 7:36 PM Page 14

52. A researcher conducts a test in which the corked anduncorked bats in table 1 are used to hit identicalpitches and reach their respective maximum swingvelocities. Which of the following is the likely result?

A. The corked bat hits the ball much fartherbecause it contacts the ball with greater velocity.

B. The corked bat hits the ball much fartherbecause the cork provides elasticity, whichdampens the bat’s resonance.

C. The balls are propelled about an equal distancebecause both bats produce near equal momen-tum.

D. The uncorked bat hits the ball much fartherbecause it has more mass.

53. A player bunts, that is, holds the bat perpendicular tothe pitch to make contact rather than swinging. Thebatter attempts to prevent the bat from rotatingaround its center of mass. If the impulse of the colli-sion creates a maximum force of 1.5 kN at a distanceof 8 cm from the center of mass, what is the maxi-mum force that the batter must exert? Assume thatthe batter’s hands act as one force at 6 cm from thecenter of mass.

A. 1.125 × 103 N

B. 1.35 × 102 N

C. 2.0 × 103 N

D. 1.5 × 103 N

54. A ball is thrown at a suspended bat at each of the fol-lowing spots on the bat. Which impact site wouldproduce the greatest vibration?

A. A

B. B

C. C

D. D

55. If a batter hits a stationary baseball, how would thevelocity of the ball after it leaves the bat change if theimpulse from the bat is the same, but the ball’s massis halved?

A. It is halved

B. It remains the same

C. It is doubled

D. Cannot be determined from the given informa-tion

56. The rubber balls used in mechanical batting cages areslightly less compressible than regular baseballs.What is the net effect on the batter’s performance?

A. The rubber ball will achieve much faster post-collision velocity.

B. The rubber ball will achieve slightly faster post-collision velocity.

C. The rubber ball will achieve slightly slowerpost-collision velocity.

D. There will be no difference in post-collisionvelocity.

BA C D

15

GO ON TO THE NEXT PAGE.

MCAT FL Test 11R 11/21/03 7:36 PM Page 15

Passage VIII (Questions 57–61)

Cathodic protection is a cost-effective means to pre-vent corrosion. Water is the major cause of corrosion inmetals, and the effect is generally more pronounced withsalt water. When designing metallic objects that will be inconstant contact with water, such as undergroundpipelines, boats, and offshore oil platforms, special caremust be taken to prevent corrosion.

The first line of defense in the battle against corrosionis coating, but this has its limitations, and few coatings arerobust enough to be relied upon on their own. One of themost cost-effective ways to protect against corrosionbesides coating is by means of cathodic protection. In sim-ple terms, this is the protection of a metal against corrosionby forcing it to become a cathode.

There are two regimes for cathodic protection. The firstis to attach to the metal structure a more easily oxidizedsacrificial metal. If these two metals are in an electrolyticsolution, they will form an electrochemical cell with themetal structure as the cathode and the sacrificial metal asthe anode. In this way, the corrosion still occurs, but at apredetermined location, and the intended structure remainsintact. Of course, the sacrificial metals must be replacedperiodically.

The second method for cathodic protection works bestin solutions with high electrical resistance and is known asimpressed current cathodic protection. It is often used toprotect steel-reinforced concrete. In this method, current issupplied from an anode via a wire to the metal. Generally,an AC power source is used and must be rectified.

Cl2 + 2e → 2Cl– 1.36 V

Ag+ + 1e → Ag 0.80 V

Cu2+ + 2e → Cu 0.34 V

O2 + 2 H2O + 4e → 4 OH– 0.40

Sn2+ + 2e → Sn –0.14 V

Fe2+ + 2e → Fe –0.44 V

Zn2+ + 2e → Zn –0.76 V

2 H2O + 2e → H2 + 2 OH– –0.83 V

Al3+ + 3e → Al –1.66 V

Mg2+ + 2e → Mg –2.37 V

Table 1–Reduction Potentials

57. Which should be used as a sacrificial metal to protectagainst the corrosion of an underwater aluminumstorage container?

A. Mg

B. Zn

C. Fe

D. Ag

58. What is the most effective method of cathodic pro-tection for an iron pipe used in transporting seawa-ter?

A. Attaching pieces of tin to the pipe.

B. Attaching pieces of zinc to the pipe.

C. Delivering an electric current to the pipe from aseparate anode.

D. Attaching pieces of silver to the pipe.

59. Which statement best explains the result of acciden-tally switching the wire connections of an impressedcurrent cathodic protection system for an under-ground storage tank?

A. There will be no effect.

B. The storage tank will corrode more quickly thanusual.

C. The system will overheat, causing weldingbetween the sacrificial metal and the tank.

D. The power source will be damaged due to ashort circuit.

60. Consider an iron ship at sea with zinc plates boltedonto its underside. Which of the following statementsis false?

A. The zinc plates will corrode faster than the ironof the ship.

B. The iron serves as the cathode in an electro-chemical cell.

C. There will be an emf of 0.32 V present.

D. The zinc plates will reduce faster than the ironof the ship.

16

GO ON TO THE NEXT PAGE.

MCAT FL Test 11R 11/21/03 7:36 PM Page 16

61. Occasionally, iron will be coated with a very thinlayer of tin to protect it from oxygen and water. Whatresults if a gap is formed in the tin coating?

A. The iron will rust more quickly than if the tinwere not present at all.

B. The iron rusts in a normal fashion in the area ofthe gap.

C. The iron will not rust because the tin serves as asacrificial anode.

D. The iron will not rust because the tin is a goodreducing agent.

17

GO ON TO THE NEXT PAGE.

MCAT FL Test 11R 11/21/03 7:36 PM Page 17

Questions 62 through 66 are NOT based on a descriptive passage.

62. Meshing the plates of a variable air capacitor will

A. increase the capacitance by increasing the areafor charge storage.

B. increase the capacitance by decreasing the resis-tance to charge flow.

C. decrease the voltage by increasing the storedcharge.

D. decrease the voltage by decreasing the capacityfor charge storage.

63. Which of the following best explains why liquidwater forms spherical globules due to surface ten-sion, in the absence of gravity?

A. hydrogen bonding

B. dispersion (London) forces

C. ionic impurities

D. gravitational forces

64. For any given substance at a pressure of 100 atm anda temperature of 30 K, which of the following state-ments is most likely accurate?

A. The equation PV = nRT is obeyed exactly.

B. The volume of the substance is slightly greaterthan that predicted by PV = nRT.

C. The volume of the substance is much less thanthat predicted by PV = nRT.

D. The van der Waals equation of state is obeyedexactly.

65. Which of the following, if added to a saturated aque-ous solution of NaCl, would most effectively reducethe chloride ion concentration?

A. INO3B. Ba(NO3)2C. K2SO4D. H2S

66. All of the following are equal to Avogadro’s numberEXCEPT:

A. the number of atoms in 11.2 liters of oxygen atSTP

B. the number of atoms in 1 mole of He at STP

C. the number of electrons in 96,485 coulombs ofelectricity

D. the number of SO42– ions in 1 liter of 1 N

H2SO4

18

GO ON TO THE NEXT PAGE.

MCAT FL Test 11R 11/21/03 7:36 PM Page 18

Passage IX (Questions 67–72)

Reduction-Oxidation reactions are used in the environ-mental industry as an aid in cleanup of hazardous sub-stances such as chlorinated hydrocarbons, the reduction oftoxicity, improvement of biodegradability, and the removalof odor and color. These reactions are helpful in remediat-ing groundwater where extensive contamination hasoccurred and have the benefits of being fairly inexpensiveand safe.

One such reaction that is used is the Fenton’s reaction.It was first described in the late 19th century and in the lastfew decades has been deployed in wastewater treatmentgroundwater treatment applications. The process relies ongenerating hydroxyl radicals using a reaction of ferrousiron and hydrogen peroxide. The hydroxyl radicals willreact with the hydrocarbons to produce carbon dioxide asa final product, and possibly chlorides, depending on whatis being treated. It requires a pH in the range of 3.0 to 5.0to work effectively, and is driven by the iron chemistry. Toapply, iron is first added in the form of a solution ofFeSO4, and then hydrogen peroxide is added slowly.

The reaction is self-replicating, as seen in the follow-ing reactions:

Fe2+ + H2O2 = –OH + Fe3+ + OH–

Reaction 1

Fe3+ + H2O2 = Fe2+ + HOO– + H+

Reaction 2

Four types of reactions take place involving the hydroxyl

radicals in water as shown in Table 1.

(1) •OH + C6H6 → (OH)C6H6•

(2) •OH + CH3OH → •CH2OH + H2O

(3) •OH + [Fe(CN)6]4– → [Fe(CN)6]3– + OH–

(4) •OH + •OH →H2O2

Table 1

67. A sample of wastewater is taken. It is determined that[OH–] = 6.0 × 10–6. What is the pH of the wastewa-ter?

A. 9.77

B. 5.22

C. 7.01

D. 10.58

68. Above a certain temperature range, usually above50°C, the efficiency of Fenton’s reaction declines.What is a possible explanation for this?

A. At higher temperatures, H2O2 decompositionincreases.

B. At higher temperatures, iron precipitation isinhibited.

C. At higher temperatures, H2O2 decompositiondecreases.

D. At higher temperatures, the activation energiesare lower.

69. What is a possible explanation for the need to add H2O2 slowly into the ground?

A. The reaction will not take place at low pH.

B. Iron precipitation to Fe(OH)3 will create oxygengas, a possibly hazardous situation.

C. Hydroxyl radicals will form in too high a con-centration to be usable.

D. Hydrogen peroxide will react with the hydrocar-bons present in the soil.

70. Which of the following statements is true of Fenton’sreaction?

A. In the first stage of the reaction, iron is oxidized.

B. 2 electrons are transferred in the second stage ofthe reaction.

C. In the second stage of the reaction, iron is oxi-dized.

D. In FeSO4, SO4 has an oxidation state of +2.

19

GO ON TO THE NEXT PAGE.

MCAT FL Test 11R 11/21/03 7:36 PM Page 19

71. What is a possible explanation for why Fenton’sreaction will not proceed quickly in basic wastewa-ter?

A. Hydroxyl formation is enhanced at pH above 5.

B. At high pH, iron catalytically decomposes theH2O2 into oxygen and water.

C. The perhydroxyl radical is a weaker oxidizerthan the hydroxyl radical.

D. At high pH, Hydroxyl radicals react with hydro-carbons to form chlorides, which prevent thereaction from continuing.

72. Which of the four hydroxyl radical reaction is pre-ferred in the remediation of groundwater?

A. 1 and 3

B. 1 and 2

C. 1, 2, and 4

D. 3 and 4

20

GO ON TO THE NEXT PAGE.

MCAT FL Test 11R 11/21/03 7:36 PM Page 20

Passage X (Questions 73-77)

While orbiting around a central massive body, astro-nomical bodies strictly obey a series of laws, which allowfor a convenient description of their orbits. These rules,derived and enumerated by Johannes Kepler, are organizedinto three separate laws.

The law of orbits: All bodies that orbit a central, mas-sive body travel in an elliptical path. The length of theellipse, or major axis, and its eccentricity, e, aid in describ-ing the geometry of the elliptical orbit; the term eccentric-ity can be thought of as the elliptical curvature of the orbit.Assuming the major axis has a length 2a, the central mas-sive body, positioned at one focus, is a distance ea from thecenter of the orbit. In a circular orbit—a special type ofelliptical orbit—the focus and center are located at thesame point. Additionally, knowing the major axis andeccentricity of an orbit enables the determination of abody’s perihelion and aphelion, when it is at its closest andfarthest position, respectively.

Figure 1

The law of areas: A line that connects a planet to thecentral massive body sweeps out equal areas in equaltimes. In figure 2, area 1 equals area 2 when ∆t1 equals ∆t2.

Figure 2

When a body is at perihelion, its potential energydecreases. Additionally, a body orbiting with a smallermajor axis corresponds to a state of less total energy.

The law of periods: The square of the period, T, of anybody is proportional to the cube of half the major axis ofits orbit.

T2 = × a3

Equation 1

Where M is the mass of the central massive body.

Conceptually, the law of periods provides a mathemat-ical relationship between an astronomical body’s periodand major axis: a body that is farther out will have a longerperiod of orbit.

73. The passage implies that the eccentricity of a circleis:

A. 0

B. 0.5

C. 1.0

D. 2.0

4π2�GM

area 1 in ∆t1

area 2 in ∆t2

-eaa

21

GO ON TO THE NEXT PAGE.

MCAT FL Test 11R 11/21/03 7:36 PM Page 21

74. Pluto has a major axis of 1.18 × 1013 m and a periodof 248 years. What is the expected major axis ofMars, which has a period of 1.88 years?

A. 2.5 × 103 m

B. 4.6 × 1011 m

C. 3.7 × 1014 m

D. 6.8 × 1021 m

75. As a planet executes an elliptical orbit about the sun,it maintains constant:

A. acceleration

B. angular velocity

C. kinetic energy

D. total energy

76. Which of the following graphs represents the forceexerted on a planet by the central body it is orbiting?

A.

B.

C.

D.

77. An astronomer discovers two new planets orbiting adistant star. If both planets have the same mass, butone orbits the star with a major axis three timesgreater than the other, which of the following must betrue?

A. The planets experience the same centripetalforce.

B. The sum of the potential energies of both plan-ets is constant.

C. The distance from a focus to the center of theorbit cannot be the same for both planets.

D. The frequency of revolution will be larger forthe closer planet.

STOP. IF YOU FINISH BEFORE TIME IS CALLED,CHECK YOUR WORK.YOU MAY GO BACK TO ANYQUESTION IN THIS SECTION ONLY.

Force

Distance from focus

Force

Distance from focus

Force

Distance from focus

Force

Distance from focus

22

MCAT FL Test 11R 11/21/03 7:36 PM Page 22

Verbal ReasoningTime: 85 MinutesQuestions 78–137

DO NOT BEGIN THIS SECTION UNTIL YOU ARE TOLD TO DO SO.

MCAT FL Test 11R 11/21/03 7:36 PM Page 23

Passage I (Questions 78–82)

It would be difficult to overstate the complexity of theJapanese language. The system of writing (or more prop-erly, systems) represents a fusion of almost entirely foreigncharacters and a spoken language so linguistically isolatedthat philologists have yet to discover a precursor. Notunlike many other ancient languages, Japanese lacked anysystem of writing at all for much of its history. Making upfor lost time, though, no fewer than three different systemsof writing are now employed.

The first Japanese system of writing was not Japanese.The kanji, a group of logographic Chinese characters eachrepresenting a word or idea, were adopted with minimalchange around the seventh century. Few languages are sogeographically close yet linguistically dissimilar. As aresult, the Japanese adopted a modified Chinese pronunci-ation for each kanji (the on-yomi) while retaining thenative Japanese spoken word that most closely fit eachkanji’s meaning (the kun-yomi). In modern Japanese, theon-yomi is used for certain kanji and the kun-yomi for oth-ers, with compound words often involving both. Furtheradding to the confusion, the Chinese language containsmany words in which variations in tone alone indicatedrastically different meanings. The adaptation of thesewords to Japanese pronunciation led to a number of homo-phones that has, without hyperbole, been called “embar-rassing” and “alarming” by scholars of the language.

The hiragana syllabary was developed in the eighthcentury by court women, who were not permitted to studykanji because they were deemed unfit to master its com-plexities. In response, they developed a simplified, flowingform of the kanji that represented all the sounds in spokenJapanese. Hiragana is phonetic rather than logographic,and is therefore far more accessible to a foreign learnerthan the kanji. Since Japanese is an open language, withnearly every syllable ending in a vowel, most consonantscannot be expressed by themselves. Hiragana is thereforenot strictly an alphabet. Katakana came about around the

same time as hiragana, also as an attempt to simplify thekanji. The sparse, angular characters correspond fairlyclosely to the hiragana and, as befitting their origin amongBuddhist monks, have a look generally considered moremasculine than hiragana, which was originally calledonnade, or “women’s hand.” The katakana have essentiallybecome the print counterparts of the “cursive” hiragana.

With so many systems jostling for position, each usedmore or less independently of the other, it would not beunreasonable to anticipate that a national movementtowards systematization of the language would settle on asingle one. A national movement was in fact started afterWorld War II; a radical idea encouraging the use of allthree systems together. A glance at any Tokyo newspaperwill reveal kanji used to represent most standard actionsand ideas, hiragana to indicate grammatical inflections andtenses, and katakana to represent adopted foreign andtechnical words, as well as to indicate emphasis. The useof the three systems has become sufficiently standardizedin this way that deviations often lend a piece of writingstrong connotations. A piece written entirely in katakana,for example, may be disconcerting to a modern reader andmay have a vaguely pre-World War II military air to it,much as a piece written all in capital letters with telegraphpunctuation might in English. While such a complex sys-tem has made the language’s learning curve high for nativespeakers and foreigners alike, it has also contributed to astunning richness of expression such that any list ofworld’s great works of art a hundred years from now willhave to be written partially in kanji, hiragana, andkatakana.

24

GO ON TO THE NEXT PAGE.

5

10

15

20

25

30

35

40

45

50

55

60

65

VERBAL REASONING

DIRECTIONS: There are ten passages in this VerbalReasoning Section. Each passage is followed by sev-eral questions. After reading a passage, select the onebest answer to each question. If you are not certain ofan answer, eliminate the alternatives that you know tobe incorrect and then select an answer from theremaining alternatives. Indicate your selection byblackening the corresponding oval on your answerdocument.

MCAT FL Test 11R 11/21/03 7:36 PM Page 24

78. The author’s explanation for the origin of Japanesehomophones relies on which of the followingassumptions?

A. The large number of homophones in the lan-guage is due to the closeness of Japanese andChinese pronunciation.

B. The presence of homophones in a language canbe considered embarrassing.

C. Spoken Japanese does not rely on tone to theextent that Chinese does.

D. Homophones are dependent on variations intone.

79. The author’s primary purpose in the passage is to:

A. argue that the Japanese language is overlycomplex.

B. describe the origins of the Japanese language’scomplexity.

C. propose a simplification in how Japanese iswritten.

D. trace the origins of logographic writing systems.

80. As used in the passage, the phrase “open language”most closely means:

A. a language that borrows systems of writing andspeaking from many different sources.

B. a language unusually open to change andreform.

C. a language in which most syllables end in vowels.

D. a language in which few consonant sounds areused.

81 According to the passage, which of the followingpieces of Japanese literature would NOT be likely tobe written entirely in katakana?

A. a modern Japanese novel

B. a list of adopted foreign words

C. a ninth-century Buddhist text

D. an early twentieth-century general’s log

82. According to the passage, the group of Buddhistmonks who developed katakana:

A. was predominantly or entirely male.

B. used hiragana as a model.

C. was considered unfit to master the complexitiesof kanji.

D. was closely involved with the military of thetime.

25

GO ON TO THE NEXT PAGE.

MCAT FL Test 11R 11/21/03 7:36 PM Page 25

Passage II (Questions 83–91)

The United States has one of the world’s oldest demo-cratic governments. The U.S. Constitution outlines thegovernment’s main features and, at more than two hundredyears of age, has required few modifications. Many expertsconsider it a marvel that the Constitution has served sowell. A comparison between the U.S. government anddemocracies abroad, many of which developed relativelyrecently, is relevant. According to the prominent constitu-tional scholar Robert A. Dahl, the U.S. government doesnot compare well in respect to the democratic ideal itself.

Dahl finds some features of the U.S. government directlyat odds with democratic principles, and others, while notundemocratic, in urgent need of reconsideration. Dahl con-siders, for example, the United States Senate. The populationof some states is nearly seventy times greater than smallerstates, yet each state is accorded two senators. A vote from acitizen of a small state weighs in effect as much as seventytimes more than a vote from a citizen of a large state. Thishardly represents an equal share in the government and there-fore contradicts a basic democratic principle. In comparison,other democratic countries also define some of their con-stituencies geographically rather than demographically, butthe resulting level of undemocratic disparity is much smaller.Advocates of the U.S. system argue that small states aregiven such disproportionate representation because theymust be able to protect their interests against larger states.Professor Dahl believes, however, that all the protection citi-zens need is enshrined in the Bill of Rights.

The next target of Dahl’s criticism is the presidency.Why does the U.S. need, he asks, another directly electedoffice? According to Dahl, Congress is invested withenough authority from voters not only to pass laws but alsoto designate an executive government. The fact that thereare two Congressional chambers already risks intra-gov-ernment conflict if different parties dominate each cham-ber. The presidency divides political power and authorityeven further. Professor Dahl argues that the claim that thepresident is the only embodiment of the political will of allthe citizens as a false crudity that originated in the mindsof presidents eager to embellish their office with undueauthority. No other democracy has adopted a presidentialsystem identical to the U.S.—a fact not surprising to Dahl.

Finally, Dahl criticizes American election rules. Similarto Great Britain, the U.S. employs the majoritarian principleof converting voters’ ballots into shares of power in govern-ment institutions and offices. According to this principle, thecandidate with the most votes in a given electoral districtreceives all of that district’s shares. This shortchangesdemocracy again. It is hardly representative to award secondand third place candidates no shares in governmental power.All citizens who did not vote for the first-place candidate are,

in effect, disenfranchised. It would be much more democraticto adopt the principle of proportionate representation, inwhich candidates who place behind the winner are awardedpower shares in proportion to the votes they received. Such agovernment would represent a greater number of voters withdifferent political views, and the government would resemblethe people who delegate their power to it.

83. Which of the following reforms of the Americanpresidency would probably be regarded as animprovement by Dahl?

A. The votes cast for the president by the citizensof smaller states would be given more weightthan those cast by the citizens of large states.

B. It would be forbidden that presidential candi-dates be recruited from among senators.

C. The president would be elected by the Congress,his choice reflecting the congressional balanceof power.

D. The president would directly establish laws.

84. What is the significance to Dahl of the fact that GreatBritain adopted a similar system of election rules asthe United States?

A. It proves that the American system has itsadvantages.

B. It supports the notion that the American and theBritish systems of government are in manyrespects alike.

C. It gives no support to the notion that the Ameri-can election of system is well designed.

D. It proves that the majoritarian election systemis, generally speaking, unpopular among demo-cratic countries.

85. If it were true that the demographic disparity betweenAmerican states was smaller two centuries ago at thetime when the rules regulating the Senate electionswere established, in which of the following wayswould Dahl most likely adjust his arguments?

A. He might predict that the undemocratic dispari-ties in countries abroad are very likely to grow.

B. He might use this fact to stress that the Americansystem of government needs reform because someof its principles have outlived their justification.

C. He might use this fact as an example of howwell the historical founders of the Americanpolitical system foresaw its future development.

D. He might argue that the Senate election lawsshould be reformed to reflect the original,milder disparities rather than the more dramaticdisparities today.

26

GO ON TO THE NEXT PAGE.

5

10

15

20

25

30

35

40

45

50

55

MCAT FL Test 11R 11/21/03 7:36 PM Page 26

86. Based on the information in the passage, what of thefollowing is likely to be Dahl’s opinion of foreignsystems of government that do not include presi-dency in the form known in the United States?

A. They are, for that reason, more democratic.

B. They lack a single representative of the will ofthe whole electorate.

C. They must have originated more recently thanthe American system.

D. They manage to better avoid the risk of adivided government.

87. Which of the following would best characterize thepurpose of paragraph one?

A. To introduce some cherished beliefs about theAmerican political system and propose ques-tions with which to analyze its qualities.

B. To portray widely held beliefs about the Ameri-can political system as false and introduce a crit-ical voice that would set them right.

C. To advocate what most Americans believe abouttheir political system despite the fact that thesebeliefs can be criticized from certain quarters.

D. To mention several characteristics of the Amer-ican political system and suggest the need to fol-low their historical development.

88. Which of the following is the main flaw of the Amer-ican presidential system, according to Dahl?

A. It causes political power to be excessivelydivided.

B. It gives individuals excessive political power.

C. Citizens who vote for the losing presidentialcandidate lack political representation.

D. It has no counterpart anywhere in the demo-cratic world.

89. In most European countries a candidate in an electionis directly nominated by a political party. In theUnited States he is usually a winner of a preliminary,so-called “primary,” election in which he runs againstthe candidates from the same party. How would thisaffect Dahl’s argument?

A. Dahl would probably concede that the Americansystem of government is one of the most demo-cratic in the world.

B. Dahl would probably concede that in somerespects the American system of elections andgovernment is more democratic than its coun-terparts in other countries.

C. Dahl would probably regard it as a further proofthat the American political system lags behindthose adopted by other democracies.

D. Dahl would probably find in it a further exam-ple of how the majoritarian principle makes itimpossible for a diversity of political views tofind representation.

90. Which of the following features of the Americangovernment criticized in the passage is NOT at oddswith the democratic principle of fair and equal repre-sentation mentioned in the passage?

A. Senate election rules.

B. The majoritarian election principle.

C. The principle of proportionate representation.

D. The popular election of the president.

91. Which of the beliefs presented in paragraph one asheld by many Americans is most strongly contra-dicted by Dahl’s arguments?

A. That the American democracy is one of theworld’s oldest.

B. That the American system of governmentunderwent few changes since its inception.

C. That the American government compares wellwith the systems of government adopted abroad.

D. That the American system of government hasserved well till the present day.

27

GO ON TO THE NEXT PAGE.

MCAT FL Test 11R 11/21/03 7:36 PM Page 27

Passage III (Questions 92–98)

William Shakespeare was no stranger to collaboration:on at least one play, The Two Noble Kinsmen, his nameappears along with that of John Fletcher. Textual criticismof the play’s vocabulary and rhythmic patterns has gener-ally verified its dual authorship, leading some scholars topropose that other plays bearing Shakespeare’s name couldalso be the work of two writers. The problem has beengreatly aggravated by recent textual and cultural studiesthat suggest collaboration was the norm, not the exception,in the late 16th to early 17th century. At present, scholarsgenerally concede that four other plays display traits of dualauthorship: Henry VIII; Timon of Athens; Pericles, Princeof Tyre; and (although more hotly debated) Edward III.(Titus Andronicus, once thought the result of collaboration,is now categorized as an early Shakespearian work.)

Pericles in particular presents scholars with both anauthorial dilemma and a textual quagmire. The play was notincluded in the First Folio, a fact that some scholars site asevidence that contemporaries knew the play was largely thework of another hand. Others point out more reasonablythat the Folio’s editors made the understandable decision toomit it only because the only available text of Pericles wasa pirated quarto of extremely poor quality. Unfortunatelythis same quarto is still our only text, and its extraordinarynumber of errors strongly suggests that it was based on“reportage”—what actors and audiences remembered fromperformances of the play. We are faced with the very realpossibility that the distinct tonal shift and changes in rhymeschemes—the critical basis for arguments of dual author-ship—weren’t in the original. Even the most casual readerwill note these variations in tone and rhyme, and can sym-pathize with scholars who refuse to believe that Shake-speare wrote the deadly dull first scenes, yet find it equallydifficult to deny his authorship of the powerful, elegiac finalscenes, so reminiscent of King Lear.

Textual evidence creates further ambiguity, suggestingthat two different publishers typeset the sheets of thepirated quarto. Surviving quarto copies of Pericles shed lit-tle light on the subject, with some clearly prose passagesset in poetic form, and with blank verse lines run togetherin a prose passage or broken into poetic lines with no con-sideration for meter.

George Wilkins’ contemporary novel The PainfullAdventures of Pericles Prince of Tyre appears to have beenbased on the play, but the opposite is equally possible. (Athird theory suggests joint inspiration and writing, compa-rable to today’s movie novelizations.) The confusing mis-prints in the quarto have forced editors to rifle throughWilkins’ novel to interpret certain passages, to the point

that “corrected” publications of Pericles (in contrast to fac-similes of the quarto) bear an even stronger resemblance tothe Wilkins’ novel, and increase the confusion.

Shakespeare’s other collaborations are generally freeof the striking shifts found in Pericles. This, together withthe textual quagmire, has fostered lively debate: Was Peri-cles written by George Wilkins and hastily revamped byShakespeare? Was it instead one of Shakespeare’s earliestefforts (perhaps contemporary with the lyrically roughHenry VI plays) revamped by an older Shakespeare with-out the energy to rewrite it entirely? Or did Shakespeare(as is perhaps the case of the earlier Timon of Athens)abandon his play midstream, and return to it later withoutrewriting the first half? If Pericles was collaboration, whyis its quality so far below that of the co-written Henry VIIIand The Two Noble Kinsmen? So far, computer analyseshave proven largely unhelpful...

92. In the context of the question of dual authorship,modern movie novelizations are most probably men-tioned in order to:

A. contrast the narrative method of RenaissanceEngland with that of today’s movie makers.

B. suggest that contemporary movie novelizationsshare many traits with works created by dualauthors.

C. highlight the similarities between the produc-tion of plays in the late 16th/early 17th centuriesand the production of films in the 20th and early21st centuries.

D. give further credence to the possibility that Per-icles is a work of dual authorship.

93. Which of the following would most challenge anassertion made in the passage concerning Periclesand the First Folio?

A. The editors of the First Folio failed to employmodern methods of textual criticism whenassembling the First Folio.

B. The quality of the play’s poetry, and not its tex-tual source, was the primary basis on which theeditors of the First Folio omitted plays from thecollection.

C. The editors of the First Folio regularly utilizedtexts based on reportage when assembling theFirst Folio.

D. Textual computer analyses of The Two NobleKinsmen, which was not in the First Folio, haveshown that Shakespeare wrote at least four of itsscenes.

28

GO ON TO THE NEXT PAGE.

5

10

15

20

25

30

35

40

45

50

55

60

65

MCAT FL Test 11R 11/21/03 7:36 PM Page 28

94. The passage cites which of the following as evidencethat Pericles, Prince of Tyre was NOT the produce ofdual authorship?

A. The difficulties in assembling a reliable textbased on reportage.

B. A literary comparison with George Wilkins’novel on the same subject.

C. The decision by Shakespeare’s contemporariesnot to include Pericles in the First Folio.

D. The incidence of blank verse in Pericles printedwith no consideration for meter.

95. It can be inferred from the passage that Titus Andron-icus most likely has which of the following charac-teristics?

A. A vitiated text based largely upon reportage.

B. A cohesive text lacking substantial tonal shiftsbetween scenes.

C. A problematical text providing modern editorswith a textual quagmire.

D. A clear text from the First Folio based on anoriginal manuscript.

96. Assume that a scholar is attempting to determinewhether Edward III shows traits of dual authorshipby Shakespeare and another writer. Based on theinformation in the passage, which of the followingwould be LEAST useful in this endeavor?

A. An original edition of the first publication ofEdward III in quarto or folio form.

B. A textual study of the language and versificationof Edward III.

C. A comparative study of a novel published atabout the same time as Edward III.

D. A textual comparison of Edward III with otherplays known to be by Shakespeare.

97. Regarding the concept of dual authorship, the authorsuggests that:

A. Works of dual authorship were rarely publishedin the collected works of authors, such as theFirst Folio.

B. Studies of quarto publications of 17th centuryplays can provide definitive answers as towhether or not they are the work of two authors.

C. While works of dual authorship could occasion-ally achieve high poetic quality, most works bydual authors were considerably inferior to KingLear.

D. Some 17th century plays credited to a singleauthor may have been written by two authors.

98. The passage implies that a play of unknown originwritten in the early 17th century could be regarded aswritten, or partly written, by Shakespeare if:

A. it closely resembles a novel published in theearly 17th century .

B. its poetic qualities are equal to or greater thanthose found in Henry VIII and The Two NobleKinsmen.

C. its rhyme scheme and other elements of versifi-cation are similar to those found in King Lear.

D. its rhyme scheme and other elements of versifi-cation are consistent throughout the play, show-ing no major tonal shifts.

29

GO ON TO THE NEXT PAGE.

MCAT FL Test 11R 11/21/03 7:36 PM Page 29

Passage IV (Questions 99–103)

In his treatise “On the Precautions that PhysiciansMust Observe” (c.1300 CE), Arnauld de Villeneuve, a dis-tinguished alchemist and physician whose patientsincluded kings and popes, provides striking insight into themedieval European physician-patient relationship. In sharpcontrast to the trust existing (or at least desired) betweenpatient and physician today, these early physicians wereapparently frequently the victims of tricks intended to testtheir abilities. The limited capabilities of medieval physi-cians, combined with this fear of trickery, led to the devel-opment of the underhanded methods enumerated byArnauld.

In the first two precautions, Arnauld tells fellow physi-cians how to avoid falling for tricks played by messengerswho brought urine samples. This example clearly showsthat physicians feared being made fools of. Analysis anddiagnosis would be difficult if the physician could not beconfident of the source of the samples. Arnauld outlinesseveral methods for careful questioning of the messengerto determine whether an animal rather than the patient pro-duced the sample. He also outlines how a physician couldslyly question the messenger to ascertain the gender andage of the patient, and the duration of the illness—all thewhile purporting to be able to glean this information fromthe sample itself.

These facts could not be determined through any pro-cedure available to medieval physicians. The one conclu-sive test available to medieval physicians was to taste theurine for sweetness, which would indicate diabetes.Despite their inability to glean other information from theurine, medieval physicians evidently pretended they could.In doing so, they set up a situation in which patients wereright to doubt physicians.

In the 7th and 17th precautions, Arnauld purports toexplain how to overcome patients’ doubts of the physi-cian’s ability. The physician who is uncertain of the diag-nosis is advised to make something up, using complicatedand arcane vocabulary like “liver obstruction,” so that thepatient and the family would not know how to dispute thediagnosis. Arnauld also urges physicians to fabricateexplanations and new treatments, not for the sake of curingthe patient, but rather to enhance the physician’s dignityand stature. He says in the 8th precaution that the reasonfor infertility is that the wife cannot hold the man’s sperm,although he had no basis for such a conclusion. In the 16th,he says that a new treatment should be devised each timethe patient is seen so that the physician appears to be ableto work without his reference books.

This type of interaction does not get the doctor anycloser to the actual cause or cure of his patient’s disease.Rather, it fosters resentment of the exclusiveness of thephysician’s knowledge, and helps to breed further distrustof medical learning and methods. It is no wonder that inlater times many called for the translation of books ofmedicine into the vernacular so that lay people had accessto some of the physician’s learning.

The third section of “Precautions” furnishes otherideas for garnering the patient’s confidence. According tothis section, even if doctors are unable to make a firm diag-nosis, they can still impress the patient if they can predictthe patient’s symptoms and discomforts. It also instructsdoctors to be polite, helpful, and diligent. This third sec-tion was, in fact, probably not written by the same author,since it takes such an entirely different approach to gainingthe patient’s trust.

99. How does the author of this passage explain thechange in tone of third section of “On the PrecautionsThat Physicians Must Observe?”

A. Arnauld of Villanova changed his views on thephysician-patient relationship over the course ofhis career.

B. The two sections were written by differentauthors.

C. It shows the evolution of the physician-patientrelationship from medieval and modern times.

D. The change of tone was not explained.

30

GO ON TO THE NEXT PAGE.

5

10

15

20

25

30

35

40

45

50

55

60

65

MCAT FL Test 11R 11/21/03 7:36 PM Page 30

100. The author of this passage believes medical bookswere translated from Latin into common languagesafter Arnauld’s time because:

A. lay people wished to understand the exclusiveLatin vocabulary used by physicians.

B. Latin was not used outside of churches after themiddle ages.

C. it facilitated teaching of new physicians.

D. physicians found that the use of Latin impededcommunication with patients.

101. How does the author feel about Arnauld as a physician?

A. The author disagrees with Arnauld’s methods,but commends his thorough knowledge ofmedicine.

B. The author admires the physician-patient rela-tionship as developed by Arnauld.

C. The author does not comment on Arnauld’s skillat healing patients, but disapproves of the meth-ods Arnauld uses to communicate with patients.

D. The author believes that Arnauld had limitedcapabilities as compared with other physiciansof his day.

102. Which of the following observations, if true, wouldmost seriously contradict the author’s opinion aboutmodern medicine?

A. When talking to modern patients, doctors delib-erately use complex medical terminology thatpatients can’t understand.

B. Modern laboratories are capable of determiningwhether urine samples actually come from ani-mals rather than patients.

C. Nurses routinely make disparaging remarksabout patients to other nurses.

D. Modern patients don’t try to trick physicians totest their competence.

103. According to Arnauld, why should physicians usedifficult vocabulary when explaining a diagnosis to apatient?

A. Without using the medical vocabulary, doctorscannot adequately explain their diagnosis.

B. The use of complicated vocabulary protects thephysician from patients who may second-guessa diagnosis.

C. Arnauld believed that lying to patients is theonly way to get them to be compliant.

D. Arnauld believed that medical vocabularyshould be translated into the vernacular.

31

GO ON TO THE NEXT PAGE.

MCAT FL Test 11R 11/21/03 7:36 PM Page 31

Passage V (Questions 104–109)

The Bedouins and other desert civilizations have longknown that if someone unfamiliar with the desert becomeslost he or she will likely never escape. This is usually notdue to the immensity of the wilderness. Rather, someonetrying to walk in a straight line without visual cues willstray slightly to the left or the right with every step; ulti-mately walking in enormous circles.

A similar problem met seafaring civilizations as theybegan to explore the great oceans. Ship captains, with noreliable method of measuring their progress or direction,could find themselves days off course. Sometimes this ledto a revolutionary discovery, such as Columbus’ stumbleupon the new world. More often than not, however, it ledto shipwreck.

As the importance of navies peaked in the 17th and18th centuries, they prompted a vigorous search for amethod of determining longitude. Since the earth rotateson longitudinal axes, most methods proposed the use oftime. If a ship’s captain knew both the local time (easilydeducible from the sun) and the time at the journey’s start-ing point, he could calculate the distance between the shipand its home port.

Two methods showed promise by the turn of the eigh-teenth century. The first, preferred by most contemporaryscientists, proposed using the placement of the moon rela-tive to the stars to determine time. If he knew the time atwhich the moon intersected a star’s path in the home port,the ship’s captain could record the time at which the moonintersected the same star over the ship and calculate thetime difference and the longitude. The universe, advocatesof this method reasoned, operated in fixed patterns like aperfect clock.

This was not practical, however. This “lunar method”required that the position of the moon at any given time beknown. Later it was discovered that the moon’s motionaround the earth operates on an eighteen year cycle, mak-ing observations that spanned shorter time periods worth-less. Furthermore a detailed knowledge of the stars in thenorthern or southern hemisphere was required. If the skywas overcast or stormy, the method could not be used at all.

Many regarded the second method as superior in the-ory but impossible in practice—the development of anaccurate ship’s timepiece. Conventional ships’ clocksgained or lost as many as fifteen minutes a day due to vari-ations in temperature and humidity. An error of even twominutes on a voyage would lead to an error of half a lon-gitudinal degree, or about thirty miles near the equator.

The development of an adequate timepiece, however,proved to be no more difficult than cataloguing lunarmotion. By the mid seventeenth century the watchmakerJohn Harrison had developed for Britain a timepiece thatlost less than one second per month at sea. Impervious tofluctuations in temperature or humidity, it could be used atall times as long as its delicate inner workings were keptseparate from the magnetic pull of the ship’s compasses.Copies of Harrison’s timepiece accompanied many of thenineteenth century’s most important voyages. This methodalso has its problems. For many years a well made time-piece proved too expensive to compete with the muchcheaper lunar tables, which had by this time been wellenough developed for poorer ship captains to look at theirclock from the inside out.

104. Which of the following, if true, would mostWEAKEN arguments in favor of the lunar distancemethod of reckoning longitude?

A. Most ship captains knew positions of stars with-out reliance on star charts.

B. A timepiece suitable for navigation is able tooperate without astronomical calibration.

C. The time of revolution of the moon around theearth is subject to variations of several minutesover a two thousand year cycle.

D. Prior to the eighteenth century, lunar measure-ments had never been recorded for longer thantwo decades at a time.

32

GO ON TO THE NEXT PAGE.

5

10

15

20

25

30

35

40

45

50

55

60

MCAT FL Test 11R 11/21/03 7:36 PM Page 32

105. Which of the following can be inferred from thepassage?

A. The lunar position method for measuring longi-tude was at least as difficult to develop as anaccurate clock.

B. The lunar position method for measuring longi-tude proved to be superior for general naval use.

C. Recording lunar positions was impossible toperform with accuracy until the late eighteenthcentury.

D. The lunar position method for measuring longi-tude was hypothesized earlier than the timepiecemethod.

106. By the mid eighteenth century, British astronomershad developed detailed catalogs of stars and lunarmovements from their observatories in London. Shipcaptains used these charts when utilizing the lunardistance method at sea. Based on this informationand the information given in the passage, the use ofan accurate timepiece would most likely be used in:

A. voyages in the northern hemisphere.

B. voyages in the southern hemisphere.

C. voyages in the eastern hemisphere.

D. voyages in the western hemisphere.

107. An early method of gauging longitude called deadreckoning involved dropping a log off a ship andmeasuring the ship’s speed by the length of time ittook the log to pass the ship. Based on the passage,the author would most likely regard this method as:

A. based on assumptions similar to those made bythe lunar distance method but not the timepiecemethod.

B. based on assumptions similar to those made bythe timepiece method but not the lunar distancemethod.

C. based on assumptions different than those madeby both the timepiece and lunar distance methods.

D. based on assumptions similar to those made byboth the timepiece and lunar distance methods.

108. Suppose that it were discovered that storms at seacaused magnetic fluctuations in the atmosphere. Thiswould most likely:

A. weaken the lunar distance method by providingan alternative method of measurement.

B. weaken the lunar distance method because of itsreduced practicality.

C. weaken the timepiece method because of itsreduced practicality.

D. weaken the timepiece method by providing analternative method of measurement.

109. Judging by the context, the phrase “poorer ship cap-tains who had to look at their clock from the insideout” most nearly means:

A. early timepieces were prone to breakage and tooexpensive to easily repair.

B. captains unable to afford timepieces used themoon and star charts to judge longitude.

C. some captains were forced to rely on the lunarmethod before the development of accuratetimepieces.

D. the lunar method supplanted the timepiecemethod as the most popular means of determin-ing longitude.

33

GO ON TO THE NEXT PAGE.

MCAT FL Test 11R 11/21/03 7:36 PM Page 33

Passage VI (Questions 110–114)

At first glance, the novelist Henry James and thephilosopher William James, though brothers, have little incommon, personally or creatively. From their earliestunconventional upbringing and education—on both sidesof the Atlantic—the boys seem to have moved in radicallydifferent directions. Both writers of great talent, they dif-fered in disposition, temperament, and philosophy. In theircorrespondence they puzzle over each other’s positionsand actions. A closer analysis of their work, however,reveals that they did indeed share some intellectual com-mon ground.

Preferring observation to action, Henry James lived alife of thought. Internal debate was sufficient stimulationirrespective of whether it led to action. In describing hischaracters’ consciousness and nature, he also, perhapsunintentionally, revealed his own. Henry’s focus on char-acters’ inner lives led to criticism of his literature for beingponderous and too heavily descriptive. In WashingtonSquare, for instance, Henry develops his heroine, Cather-ine Sloper’s inner conscience and thought process intremendous detail, often pages at a time. Henry also devel-oped his novels through reflection on his own circum-stance as an American expatriate in Europe. Contrary tohis chosen solitary life, Henry was fascinated by the natureof social interaction, especially social and cultural restric-tions on the individual. This is particularly apparent in thecultural clash between American outsiders and upper-classEuropean society rendered in such works as Daisy Millerand Portrait of a Lady.

Henry’s older brother William James, on the otherhand, though troubled by physical ailments throughout hislife, was fascinated by humanity’s freedom to act, and theramifications of that freedom. His refusal to believe thathis frail health was predetermined and insurmountable ledto his interest in the notion of free will. A scientist by train-ing, William subjected his contemporaries’ notions of theuniverse, the will, and religious experience to strict empir-ical analysis and methodical inquiry. His groundbreakingwork in psychology emphasized the material and biologi-cal over the traditional theological view of his day: con-sciousness, which held primacy in his brother’s work, wasfor William only the subjective experience of the physicalactivities of the brain. Emotional states followed physio-logical responses, not vice versa. When William expandedhis interests to include questions of religious experienceand philosophical inquiry, he developed the theory ofmethod that came to be called Pragmatism, in which thetruth of any belief or value was inseparable from its prac-tical consequences—in other words, an idea was only trueso far as it had utility and could be put into practice.

Though often to wildly different results, Henry andWilliam James both focused on the mind. The ideas oftheir father, Henry James, Sr., formed the basis for theirpursuit of knowledge. The elder James stressed to his chil-dren the importance of constructing their philosophicaland intellectual worldviews on reason rather than faith ortradition, leading them to reject, for example, previouslyaccepted notions of the impetus to action (or, in Henry’scase, inaction). In addition, each independently formulatedthe notion of the “stream of consciousness,” taken forgranted today: that the mind is not an orderly chamberthrough which thoughts march in linear fashion, presidedover by reason, as was generally believed, but rather, thatthought proceeds in a constant, disorderly manner from theexperiences, perceptions, and emotions of the individual.In both brothers’ view, from this stream flows one’s indi-vidual identity.

110. Based on the ideas contained in the passage, the factthat Henry and William independently arrived at thenotion of the “stream of consciousness” suggeststhat:

A. neither of the men took seriously the ideas oftheir father, who valued reason.

B. the brothers, bot well read, were familiar withthe same research publications.

C. the James family was unusual in that their mindsprocessed ideas in orderly, linear ways.

D. some aspect of their intellectual background ledeach of them to reach the same conclusion.

34

GO ON TO THE NEXT PAGE.

5

10

15

20

25

30

35

40

45

50

55

60

65

70

MCAT FL Test 11R 11/21/03 7:36 PM Page 34

111. If the author is correct in his assessment, HenryJames would most likely have agreed with all of thefollowing ideas EXCEPT:

A. an upper class American of James’s day wouldhave had trouble fitting into European society.

B. an individual’s identity is based on his percep-tions, experiences, and feelings.

C. the most productive way to gain knowledge isthrough methodical analysis under laboratoryconditions.

D. society can be explored without interaction.

112. Which of the following best expresses the author’sview of the relationship between literature and scien-tific research?

A. Literature and science cannot be comparedbecause they inevitably are concerned withcompletely different kinds of knowledge.

B. Literary works reveal more about their authorsthan scientific works do.

C. Literature and scientific research can eachreflect personal concerns.

D. Literature and science are ultimately differentbranches of the same approach to knowledge.

113. Given Henry James’s interests and concerns asdescribed in the passage, what was likely to havebeen the point of his exploration of “the culturalclash between American outsiders and upper-classEuropean society”?

A. To show that Americans can easily assimilateinto European society, since their ancestorsprobably came from there.

B. To show that, because of their training andupbringing, Americans would have been limitedin their social interactions in European society.

C. To argue in favor of more understandingbetween people of different backgrounds andcultures.

D. To show that expatriate Americans in Europewould have been limited to the role of observersof society in their adopted homes.

114. If William James conducted a survey of people’semotional states, which of the following would provehis theories mentioned in the passage?

I. After nearly drowning in a boating acci-dent, the subject became extremely upsetwhen he came into contact with water.

II. Grieving over the loss of a loved one, thesubject refuses to eat and becomes frail.

III. The subject is almost always content,regardless of his surroundings.

A. I only.

B. I and II.

C. III only

D. II and III.

35

GO ON TO THE NEXT PAGE.

MCAT FL Test 11R 11/21/03 7:36 PM Page 35

Passage VII (Questions 115–120)

Science and language have coexisted uneasily since thebiblical Tower of Babel fragmented language’s unity, andby necessity, its reasonableness. The classic scientist seekssimplicity in the action of the universe and in its descrip-tion; language intrudes, introducing ambiguity and error.Still, things must be explained and described; thus,researchers have generally accepted language as necessarywhile simultaneously lamenting its imprecision.

Scientists have generally acted somewhere along acontinuum of two extremes when dealing with the prob-lem of language—constructing a new “perfect language”or giving up the idea of meaning in words. Attempts toconstruct a “perfect language”, universally spoken andapplicable, have been ambitious and admirable, if imprac-tical. John Wilkins, a seventeenth century polymath notedfor his theories about the moon, the construction of trans-parent beehives, and perpetual motion machines,attempted to systematize language by dividing the uni-verse into forty classes of objects, each assigned a uniquemonosyllable. Wilkins divided these classes into “differ-ences,” with each difference assigned a consonant.Finally, each difference was divided into “species,” eachof which was assigned a unique vowel. The three werethen combined into a word in the order class-difference-species. By making language scientific, Wilkins hoped tomake it safe for science: any word in this perfect languagewould contain its own definition.

Systematic languages tend to collapse under their ownweight. Though certain pockets of systematic languagehave flourished (as international chemists’ conventionsdemonstrate), the sheer inertia of culture prevents any lan-guage, reasonable or otherwise, from being adopted enmasse on merit alone. Faced with the fact that the ideal isunattainable, and finding themselves in a newly demo-cratic world where scientific dialogue has been absorbedinto the general cultural conversation, many scientists havegiven up the struggle for meaning altogether.

Abject surrender poses few problems if the thing beingdescribed can be understood apart from words. While themetaphor might have been necessary for the Greeks, themodern zoo-going public loses little by no longer knowingthat a hippopotamus is a “water horse,” or that a coelacanthis a hollow-spined fish. The image of the thing is accessi-ble, and so words are not needed to bridge the gap betweenthe thing and its conception. With such easy public accessto the wonders of the world, a scientist can indulge in nar-cissistic naming with few negative repercussions. As longas it is on public display in zoo or sky, what harm if thefinch is named after one’s mother or the star after one’s

favorite dessert, rather than according to a dull, if informa-tive, system.

Scientific culture has become comfortable with thislazy naming, a relic from the days when a new discoverycould be stuffed with sawdust or pickled in formaldehydeand packed off to the British Museum for public consump-tion. But most things discovered today are decidedly dif-ferent: an atom cannot hold sawdust, and pickling a geneis impossible. Still, though, the capitulation to language’sirrationality continues, and it does so at the expense ofunderstanding. Genes are named after cartoon hedgehogs;the mysterious subatomic quark is strange, up, down, top,bottom, charmed, and as abstract as if it had no name at allbecause its discoverers valued cuteness above description.Modern discoveries such as these are not presented forviewing to the public, as the hippopotamus is at the zoo,and therefore cannot rely on image as understanding.While there may be no place for a John Wilkins in modernmicroscopic and submicroscopic science, there is certainlyroom for names that acknowledge nature’s fundamentalunity and take steps towards communicating that unity to asophisticated public.

115. With which of the following conclusions about mod-ern science would the author most likely NOT agree?

A. Most new discoveries in modern scientific disci-plines are unable to be visually verified.

B. “Perfect languages” are impractical when deal-ing with discoveries that aren’t visible.

C. Language is necessary in order to adequatelydescribe submicroscopic phenomena.

D. Modern scientists are too eager to change thename of organisms to something less meaning-ful but more whimsical.

36

GO ON TO THE NEXT PAGE.

5

10

15

20

25

30

35

40

45

50

55

60

65

70

MCAT FL Test 11R 11/21/03 7:36 PM Page 36

116. The author most likely refers to the quark as“strange” (line 67) in order to:

A. demonstrate that most modern discoveries areso unusual that only descriptive names will fit.

B. argue that most modern discoveries are too oddto merit a descriptive name.

C. criticize the fact that many modern discoveriesare given names that are not descriptive.

D. provide an example of a name that effectivelydescribes a difficult-to-understand phenomenon.

117. Which of the following, if true, would mostWEAKEN the author’s argument about trends in sci-entific naming?

A. Many systems of classification in the past haveused modified versions of “perfect languages”that included both systematic and nonsystematicelements.

B. Most submicroscopic phenomena behave inways that have no visible analogy.

C. Most Greeks had never seen either a live hip-popotamus or a picture of one.

D. Nearly all natural phenomena can be describedusing terms that can also effectively describevery different phenomena.

118. Suppose a scientist who discovered a gene respon-sible for building muscle mass wanted to name hisdiscovery using a classification similar in structureto John Wilkins’ system. According to the passage,which of the following could be a valid name forthe gene?

A. Hercules

B. Wilkin’s Gene

C. Muscle-Growing Factor

D. Reba

119. According to the passage, all of the following wouldrequire a descriptive name EXCEPT:

A. an exotic species of orchid.

B. a cellular protein with an unknown function.

C. a newly-discovered dinosaur species.

D. a recently discovered prehistoric tribe ofhumans.

120. Suppose that in the distant future a new microscopicmethod was developed that allowed the public to eas-ily view subatomic particles in museums. This newmethod would:

A. weaken the author’s argument that the currentnaming system for quarks is inappropriate.

B. weaken the author’s argument that new discov-eries should be given descriptive names.

C. strengthen the author’s argument that new dis-coveries should be made available to the publicas soon as possible.

D. have no effect on the author’s argument.

37

GO ON TO THE NEXT PAGE.

MCAT FL Test 11R 11/21/03 7:36 PM Page 37

Passage VIII (Questions 121–126)

Peter Gay’s book, The Education of the Senses, re-examines Victorian bourgeois attitudes about sensualityand sexuality in an attempt to discredit the pervasive andnegative view of the Victorian bourgeois as repressed andrepressive people whose outward prim public appearancesoften hypocritically masked inner lascivious thoughts andprivate behaviors. One of the most interesting facets ofGay’s study is his discussion of the necessary, yet taboo,issue of birth control during the latter part of the nine-teenth century.

Gay points out that the very process of giving birth wasdangerous to both the newborn and the mother—that mostwomen suffered greatly during the birth, that many chil-dren and new mothers died within five years of a birth, andthat many women approached the child-bearing processwith trepidation although they believed that producing off-spring was a woman’s ultimate fulfillment. Advice orassistance from the medical profession—whether licenseddoctors or self-trained midwives—was sorely lacking andinconsistent, hardly capable of reassuring the expectantmother and father as to the woman’s and the baby’s safety.

In fact, the medical profession itself was largelyresponsible for promulgating myths and rumours about thedangers of attempting to limit family size through use ofsome form of birth control, regardless of the fact that somany women and children died each year due to compli-cations of pregnancy or birth. Some medical, religious, orsocial experts did acknowledge that the continual cycle ofbirth was not only detrimental to the health of the motherbut also could take a toll on the quality of life of the fam-ily since multiple children increased the financial burdenand responsibility of the father. Furthermore, women wereoften caught in this cycle of pregnancy and childbirth wellinto their late 40s, greatly increasing the health risks andmortality rate of mother and child alike. It seemed appro-priate and even necessary, then, to makes efforts to limitthe number of offspring in order to benefit the family unitand thus the greater good of the larger society itself.

Nonetheless, open discussion of birth control meth-ods, both natural and device-assisted, was rare, evenbetween a doctor and his patient. Most information waspassed along by word-of-mouth, which inevitably led to agreat deal of unchecked misinformation that was, at times,deadly. Gay maintains that a primary motivation for thisreticence was deeply ingrained in the Victorian bourgeoismindset that emphasized the value of family and tradi-tional roles and thus encouraged women to be produc-tive—in the very literal, procreative sense. Thoughconcerned husbands certainly did take steps to assure that

their wives and their families were not jeopardized by anoverabundance of offspring, a widespread effort to limitfamily size was not firmly rooted in society until theadvent of a strong woman’s movement, which did notmake many real and meaningful strides in changing pub-lic attitudes and behaviors until the early twentieth cen-tury. Thus Victorian bourgeois women were obliged tofulfill their societal role as child bearers despite very realfears about the toll this could take on their health and onthe well being of their family.

121. Which of the following is NOT, according to theauthor, a reason that most Victorian women did notuse any form of birth control?

A. The medical profession did not offer any meth-ods of birth control, but only vague suggestionsfor limiting the number of pregnancies a womanhad during her lifetime.

B. Although the risks to women were widely rec-ognized, women were still held responsible forcontinuing the family line.

C. Accurate information about safe forms of birthcontrol was difficult to ascertain, even from doc-tors or midwives.

D. Many medical experts were reluctant to discussbirth control with their patients because theythemselves were not well-informed about theoptions and consequences.

38

GO ON TO THE NEXT PAGE.

5

10

15

20

25

30

35

40

45

50

55

MCAT FL Test 11R 11/21/03 7:36 PM Page 38

122. Which of the following, if true, would most seriouslyweaken Gay’s argument that deeply ingrained socialattitudes were responsible for the lack of open dis-cussion about birth control?

A. The majority of Victorian women were satisfiedwith their role as wives and mothers.

B. Limiting family size was inconsistent with bour-geois perceptions about social status.

C. Bourgeois Victorians believed the ideal familyconsisted of fewer than five children.

D. Many Victorian physicians actually knew thatfamily planning would improve women’s over-all health.

123. The passage suggests that which of the followingwas/were commonly associated with childbirth dur-ing the Victorian era?

I. Death of the child.

II. The need for additional family income.

III. Appreciation of the mother’s suffering.

A. I only.

B. I and III.

C. II and III.

D. I and II.

124. Which of the following general theories would bemost consistent with Gay’s arguments as presented inthe passage?

A. Examination of socio-historical context offersthe modern historian little useful informationabout prevailing attitudes and behaviors of acertain time period.

B. Understanding the psychological basis forsocial actions can help to explain apparentlycontradictory behavioral patterns.

C. Social historians can best analyze past culturesby applying modern theories to the work of ear-lier critics.

D. Critics should not attempt to rework prior stud-ies of social classes since they cannot properlyunderstand the historical context of their prede-cessors.

125. The passage suggests that Peter Gay is LEAST likelyto agree with which of the following statements?

A. Social critics should occasionally re-examineexisting beliefs to make sure that they are appro-priate.

B. The prevailing views of Victorian bourgeoissociety were based on accurate socio-culturalperceptions.

C. The women’s movement of the early twentiethcentury helped to bring the issue of birth controlinto the public consciousness.

D. Common mindsets must be altered before realsocial change can be accomplished.

126. Based on the passage, which of the following doesthe author consider was the most important factorcontributing to the high danger associated with Vic-torian childbirth?

A. False and misleading information about therisks of childbirth.

B. Unsanitary conditions in the birthing room.

C. Successive cycles of pregnancy and childbirth.

D. Oppressive social attitudes that forced womento procreate.

39

GO ON TO THE NEXT PAGE.

MCAT FL Test 11R 11/21/03 7:36 PM Page 39

Passage IX (Questions 127–132)

Market theory, logic, common sense, and all empiricalevidence demonstrate that no one can consistently pickwinners. Unfortunately, many professionals providedefinitive (albeit meaningless) recommendations whenasked for good investments. Furthermore, because offaulty compensation schemes, financial professionals maynot only be unhelpful, but also overtly harmful.

According to market theory, every financial securitysells, right now, at exactly the right price: the highest pricewhich actual buyers are willing to pay and the lowest priceat which actual sellers are willing to sell. In the absence ofillicit, insider information, no one can have a valid basisfor believing the price will go up (or down), unless he canevaluate known information better than everyone else andhas good reason to believe others will soon re-evaluatetheir own faulty analyses.

It is, also, no secret that Wall Street firms make moneyfrom transaction fees and selling advice to the public. Inother words, they make their profits as bookies (fees) andtouts (selling advisory services), not by picking winners. Ifthese gigantic, powerful firms that run “the game” maketheir money in this fashion, is there any reason to believethat the financial advisor down the block can make moneyby picking winners?

Many financial salesmen restrict their sales pitch tostating which mutual funds have performed well in thepast. Although they quickly point out that “past results areno guarantee of future performance”, they then focus theirpresentation on past performance. Studies show that notonly is there no guarantee, but there is also no correlationbetween a fund’s past and future performance. What hap-pened yesterday is, simply, unrelated to what will happentomorrow.

Sales staffs, compensated by commissions, have avested interest in convincing clients to invest in the productsthat pay the highest commissions. Not surprisingly, there isa negative correlation between the level of commissions andthe best interests of clients. Furthermore, commissionedsalesmen are motivated to engage in more transactionsthan a client’s best interests dictate. Some promotionalmaterials given by product providers to salesmen stress thecommission to the agent rather than the benefits to theclient. Sometimes the inducements are more subtle: theyoffer an all expenses paid vacation to high volume sales-men. Others hide behind exclusivity. For example MerrillLynch Funds are only available to customers of theserespective firms. These funds charge relatively higher feesto the client and pay higher commissions to the salesman.

The mere existence of Compliance Departments is,essentially, a tacit acknowledgement that self-servingbehavior by commission-based salesmen can be a realproblem. These departments, however, often actuallyshield wrongdoers—much like Internal Affairs of PoliceDepartments have sometimes been accused of not beinghard enough on “bad cops”. Advisors compensated byasset-based fees have less ability to overtly harm clients,but they, too, can sometimes succumb to non-client cen-tered decisions. Specifically, if an advisor is paid $15,000per year to manage a $1 million dollar portfolio regardlessof how much attention he actually spends managing it, hewill be motivated to minimize the actual time spent while,simultaneously, over-exaggerating the level of attention tothe client.

People do occasionally need financial advice. In suchinstances, they should seek the services of an advisor whois compensated on an hourly basis. Even a high hourly feewould be substantially less than the commissions or asset-based fees paid to professionals who are compensated inthat manner—and the advice would come with less self-interest by the advisor.

127. From paragraph 1, all of the following can beinferred about the author’s beliefs EXCEPT:

A. clients incorrectly assume that financial profes-sionals can pick winners.

B. recommending medicines, at some level, isappropriately analogous to recommendinginvestments.

C. the ability to pick winners consistently is possi-ble, but difficult.

D. compensation schemes influence quality ofservice.

40

GO ON TO THE NEXT PAGE.

5

10

15

20

25

30

35

40

45

50

55

60

65

70

MCAT FL Test 11R 11/21/03 7:36 PM Page 40

128. According to the last paragraph, the author recom-mends hourly fees primarily because:

A. financial professionals should only be compen-sated for time spent.

B. other compensation schemes pay too much tofinancial professionals.

C. hourly fees do not distort the nature of theadvice given.

D. hourly fees give clients greater control over theirinvestments.

129. According to the information in the passage, whichof the following is most often used to justify invest-ment recommendations?

A. Market theory only.

B. Market theory, common sense, empirical evi-dence and logic.

C. Past performance only.

D. Past performance and superior knowledge.

130. Which of the following is/are cited by the author asindicative that commissions distort the advice thatfinancial professionals give?

I. Promotional giveaways by product providers.

II. The over-emphasis of historical results.

III. Financial professionals’ need to over-exag-gerate the time spent on a client’s account.

A. I only

B. I and III only

C. II only

D. II and III only

131. The author describes how Wall Street makes itsmoney in order to demonstrate that:

A. no one can pick winners.

B. the system is unfair to the small investor.

C. investing is functionally equivalent to betting onhorses.

D. Wall Street firms are superior to local financialprofessionals.

132. Which of the following observations in the passagemost directly acknowledges the potential for brokerimpropriety?

A. Brokers often recommend securities on thebasis of past results alone.

B. Wall Street makes much of its money from advi-sory services.

C. Compliance departments are set up to monitorinappropriate broker behavior.

D. Some brokerage firms offer exclusive funds notavailable to brokers of other firms.

41

GO ON TO THE NEXT PAGE.

MCAT FL Test 11R 11/21/03 7:36 PM Page 41

Passage X (Questions 133–137)

Most traditional criticism surrounding W.B. Yeats’work has as its organizing principle the poet’s relationshipto Ireland. This is to some extent sound in its reasoning:Yeats is irredeemably tied to the nation of his birth both insubject and in reputation. While Shakespeare’s Henry IVmay be first bill from Boston to Beijing—and adapt itselfentirely to each culture—Yeats’ Cathleen ni Houlihan can-not be staged without some nod to both poet and island.

Critics generally divide Yeats’ work into two majorphases: his early writing, extending roughly up to the Irishrevolution, which draws mainly from traditional Irishmythology, and his late post-revolutionary work, whichveers into more philosophical and apocalyptic territory. Inthis Hiberno-centric view, Yeats’ aesthetic trajectory fol-lowed that of an island that shifted over his lifetime froman idyllic Arcadia to a hotbed of bloody revolution andinternecine turf wars.

Proponents of this dominant view underestimate theimportance of Yeats himself. Far from being an accuratereflection of his country, the Protestant who never learnedGaelic was an outsider who made himself integral to thebirth of the Irish Republic not by articulating what it was,but by helping to define, in laureate-caliber verse, what itshould be. Many Irish scholars concerned less with litera-ture than with history will argue that Yeats was revolution-ary first, poet second.

Those who agree that Yeats was himself directly, ifonly partially, responsible for the violence that wrackedIreland, however, overlook the fact that Yeats’ foremostconcerns were aesthetic. The poet who wondered aboutCathleen ni Houlihan, “Did that play of mine send out/Certain men the English shot?” was as concerned for hiswork as for those in whom it incited rebellion. Throughouthis career, Yeats took advantage of Irish tradition when itsuited his ends, but was quick to challenge or rewrite thattradition when it did not further his intent.

It would be wrongheaded to try to separate Yeats com-pletely from Ireland. Rather than stating that Irelanddefined the poet, though, it would be more accurate toargue that Yeats created Ireland in his own image. Much ofour knowledge of Greek mythology comes from Homerbecause so little else exists; likewise, our estimation of thisnorthern island depends greatly on the poet who domi-nated that island’s literature. While the historical Irelandthat we see may not be the real Ireland of the time, it is thehistorical Ireland because Yeats made it so, and did soaccording to a vision that had little do with Ireland itself.

While there is little doubt that pre-revolutionary Ire-land was idyllic in ways, early Yeats lyrics like “The LakeIsle of Innisfree” exaggerate this tranquility. The typicalimage of rolling green hills and cheerful farmers contrastswith the reality of poverty, famine and prejudice. We seethe former image through Yeats whose own Irish experi-ence obscured the reality from him and in turn his audi-ence. While post-revolutionary Ireland was grim, darkpoems like “Nineteen Hundred and Nineteen” surely makeit more so. The lights and darks of Yeats’ poems have asmuch to do with his personal tides as with the actual his-tory. Ireland reflects Yeats: the world sees it through a sin-gle set of eyes which do not themselves focus on Ireland,but rather use it as a green lens with which to view an aes-thetic devoid of place or time. The little island holds suchsway over the world’s imagination not simply because it isbeautiful, but because we must inevitably see it throughone of history’s great imaginations.

133. A critic who evaluated Yeats’ work according to a tra-ditional approach would be most likely to agree withwhich of the following?

A. Traditional Irish mythology contains many ele-ments that can be considered idyllic.

B. Had Yeats not lived, the Irish revolution mightnot have occurred.

C. Yeats’ later poetry is too violent to accuratelyreflect the Ireland of his time.

D. Yeats could not have been a great poet had hebeen born outside of Ireland.

42

GO ON TO THE NEXT PAGE.

5

10

15

20

25

30

35

40

45

50

55

60

65

MCAT FL Test 11R 11/21/03 7:36 PM Page 42

134. Which of the following statements would both theauthor and the traditional critics most likely agreewith?

A. Great poets are generally characterized by thestyles and themes of their homeland.

B. Yeats’ early poetry was partially responsible forthe violence of the Irish Revolution.

C. Yeats’ early poetry exaggerates the simplicityand peacefulness of the pre-revolutionary Irishcountryside.

D. Yeats’ later poetry coincided with a dark periodin Irish history.

135. Based on the passage, which of the following state-ments would the author most likely NOT agree with?

A. Knowledge of Gaelic was common among Irishrevolutionaries during Yeats’ time.

B. Ireland’s literature garnered relatively littleinternational interest before the time of Yeats.

C. Yeats’ motives for inciting violence in the IrishRevolution were primarily aesthetic.

D. An accurate understanding of Yeats is impossi-ble without a working knowledge of the IrishRevolution.

136. Suppose it were discovered that throughout the IrishRevolution, Yeats wrote significant amounts of pro-paganda in order to increase support for the revolu-tion among the peasantry. This would:

A. contradict the author’s point that Yeats neverlearned Gaelic.

B. weaken the author’s argument that Yeats wasprimarily concerned with the aesthetics of hiswork.

C. weaken the author’s argument that Yeats wasonly partially responsible for the violence of therevolution.

D. weaken the author’s argument that Yeats’ “lateperiod” contained apocalyptic elements.

137. Many of Yeats’ later poems were based on philo-sophical cycles which he created to explain history.This would most likely affect the author’s argumentin all of the following ways EXCEPT:

A. further demonstrate the importance of aestheticsover historical accuracy.

B. further explain why Yeats disregarded Ireland’spre-revolutionary poverty in favor of an idyllicportrait.

C. further demonstrate how Yeats’ Ireland repre-sented his own imagination.

D. further explain the lights and darks in his laterpoems.

STOP. IF YOU FINISH BEFORE TIME IS CALLED,CHECK YOUR WORK.YOU MAY GO BACK TO ANYQUESTION IN THIS SECTION ONLY.

43

MCAT FL Test 11R 11/21/03 7:36 PM Page 43

MCAT FL Test 11R 11/21/03 7:36 PM Page 44

Writing SampleTime: 60 Minutes

2 Prompts, Separately Timed:30 Minutes Each

DO NOT BEGIN THIS SECTION UNTIL YOU ARE TOLD TO DO SO.

MCAT FL Test 11R 11/21/03 7:36 PM Page 45

WRITING SAMPLE

DIRECTIONS: This section is a test of your writing skills. Thesection contains two parts. You will have 30 minutes to completeeach part.

Your responses to the prompts given in the Writing Sample willbe written in the ANSWER DOCUMENT.Your response to Part 1must be written only on the answer sheets marked "1," and yourresponse to Part 2 must be written only on the answer sheetsmarked "2." You may work only on Part 1 during the first 30 min-utes of the test and only on Part 2 during the second 30 minutes.If you finish writing on Part 1 before the time is up, you mayreview your work on that part, but do not begin writing on Part 2.If you finish writing on Part 2 before the time is up, you mayreview your work only on Part 2.

Use your time efficiently. Before you begin writing a response,read the assignment carefully and make sure you understandexactly what you are being asked to do. You may use the spacebelow each writing assignment to make notes in planning yourresponses.

Because this is a test of your writing skills, your response to eachpart should be an essay composed of complete sentences andparagraphs, as well organized and clearly written as you canmake it in the allotted time. You may make corrections or addi-tions neatly between the lines of your responses, but do not writein the margins of the answer booklet.

There are six pages in your answer booklet to write yourresponses, three pages for each part of the test. You are notrequired to use all of the pages, but to be sure that you haveenough space for each essay, do not skip lines.

Essays that are illegible cannot be scored. In addition, essaysthat are not written in English will not be scored.

46

MCAT FL Test 11R 11/21/03 7:36 PM Page 46

PART 1

Consider the following statement:

A good leader allows everyone to share responsibility.

Write a unified essay in which you perform the following tasks. Explain what you think the above statement means. Describea specific situation in which a good leader might not allow everyone to share responsibility. Discuss what you think determineswhen good leadership means allowing everyone to share responsibility and when it does not.

47

MCAT FL Test 11R 11/21/03 7:36 PM Page 47

PART 2

Consider the following statement:

When a country has no enemies, it must invent them to maintain its national identity.

Write a unified essay in which you perform the following tasks. Explain what you think the above statement means. Describea specific situation in which a country might not need enemies to maintain its identity. Discuss what you think determineswhether identifying enemies is necessary for a country to maintain its identity.

48

MCAT FL Test 11R 11/21/03 7:36 PM Page 48

Biological SciencesTime: 100 MinutesQuestions 138–214

DO NOT BEGIN THIS SECTION UNTIL YOU ARE TOLD TO DO SO.

MCAT FL Test 11R 11/21/03 7:36 PM Page 49

50

BIOLOGICAL SCIENCES

DIRECTIONS: Most of the questions in the BiologicalSciences test are organized into groups, with a descrip-tive passage preceding each group of questions. Studythe passage, then select the single best answer to eachquestion in the group. Some of the questions are notbased on a descriptive passage; you must also selectthe best answer to these questions. If you are unsure ofthe best answer, eliminate the choices that you knoware incorrect, then select an answer from the choicesthat remain. Indicate your selection by blackening thecorresponding circle on your answer sheet. A periodictable is provided below for your use with the questions.

1

H

1.0

2

He

4.0

3

Li

6.9

4

Be

9.0

5

B

10.8

6

C

12.0

7

N

14.0

8

O

16.0

9

F

19.0

10

Ne

20.2

11

Na

23.0

12

Mg

24.3

13

Al

27.0

14

Si

28.1

15

P

31.0

16

S

32.1

17

Cl

35.5

18

Ar

39.9

19

K

39.1

20

Ca

40.1

21

Sc

45.0

22

Ti

47.9

23

V

50.9

24

Cr

52.0

25

Mn

54.9

26

Fe

55.8

27

Co

58.9

28

Ni

58.7

29

Cu

63.5

30

Zn

65.4

31

Ga

69.7

32

Ge

72.6

33

As

74.9

34

Se

79.0

35

Br

79.9

36

Kr

83.8

37

Rb

85.5

38

Sr

87.6

39

Y

88.9

40

Zr

91.2

41

Nb

92.9

42

Mo

95.9

43

Tc

(98)

44

Ru

101.1

45

Rh

102.9

46

Pd

106.4

47

Ag

107.9

48

Cd

112.4

49

In

114.8

50

Sn

118.7

51

Sb

121.8

52

Te

127.6

53

I

126.9

54

Xe

131.3

55

Cs

132.9

56

Ba

137.3

57

La *

138.9

72

Hf

178.5

73

Ta

180.9

74

W

183.9

75

Re

186.2

76

Os

190.2

77

Ir

192.2

78

Pt

195.1

79

Au

197.0

80

Hg

200.6

81

Tl

204.4

82

Pb

207.2

83

Bi

209.0

84

Po

(209)

85

At

(210)

86

Rn

(222)

87

Fr

(223)

88

Ra

226.0

89

Ac †

227.0

104

Rf

(261)

105

Ha

(262)

106

Unh

(263)

107

Uns

(262)

108

Uno

(265)

109

Une

(267)

*

58

Ce

140.1

59

Pr

140.9

60

Nd

144.2

61

Pm

(145)

62

Sm

150.4

63

Eu

152.0

64

Gd

157.3

65

Tb

158.9

66

Dy

162.5

67

Ho

164.9

68

Er

167.3

69

Tm

168.9

70

Yb

173.0

71

Lu

175.0

90

Th

232.0

91

Pa

(231)

92

U

238.0

93

Np

(237)

94

Pu

(244)

95

Am

(243)

96

Cm

(247)

97

Bk

(247)

98

Cf

(251)

99

Es

(252)

100

Fm

(257)

101

Md

(258)

102

No

(259)

103

Lr

(260)

PERIODIC TABLE OF THE ELEMENTS

MCAT FL Test 11R 11/21/03 7:36 PM Page 50

Passage I (Questions 138–144)

Among the most well understood neurotransmitters isa group known as the catecholamines. Each of thesemolecules contains a catechol nucleus—a benzene ringwith two hydroxyl substituents—and an amine. Dopamineand its derivatives norepinephrine and epinephrine arecommon catecholamines.

Dopamine (DA) is largely produced and used in thebrain. While its functions have only begun to be explored,researchers have already found this neurotransmitter to beinfluential to both emotional balance and motor coordina-tion. Both depression and Parkinson’s disease are nowassociated with an imbalance of the brain’s dopamine.

Dopamine is synthesized in the brain as follows: First,the amino acid tyrosine crosses from the blood into thebrain across a semi-permeable membrane that separatesthe two called the blood-brain barrier. A series of enzymestransforms this tyrosine molecule into dopamine as isshown in figure 1. Tyrosine hydroxylase (TH) adds a sec-ond (–OH) group to the ring, ortho to the first, to create L-DOPA (dihydroxyphenylalanine). Studies have shown thatdoubling the concentration of TH doubles the rate ofdopamine synthesis. Next AAAD (aromatic L-amino aciddecarboxylase) replaces the carboxyl group with hydro-gen, freeing a molecule of CO2 and leaving dopamine. Theneurotransmitter is then taken up into vesicles. Once in thevesicles DA can remain unchanged or can be further mod-ified to create derivative neurotransmitters. For example,some vesicles contain an enzyme called dopamine betahydroxylase (DβH), which converts dopamine into nore-pinephrine.

The vesicles are stored in the neuron’s synaptic termi-nal and are released exocytotically when an action poten-tial travels down the axon and reaches the terminal. Themolecules inside the vesicle are emptied into the synapticcleft and activate receptors on the postsynaptic cell. In thecase of dopamine, the postsynaptic effect is excitatory andthe second cell depolarizes slightly. The neurotransmittermust then be cleared from the synapse.

The dopamine in the synaptic cleft is drawn back intothe presynaptic cell and other cells by reuptake proteins.Dopamine in the cytoplasm is brought into the mitochon-dria where Mono Amine Oxidase (MAO) modifies it, mak-ing it biologically inactive. Aldehyde dehydrogenasefurther transforms the resulting molecule to DOPAC (3,4-dihydroxyphenyl acetic acid).

Figure 1

NH2

CH

C

H2C

OH

O

HO

Tyrosinehydroxylase

NH2

CH

C

H2C

OH

O

HO

HO

NH2

CH2

H2CHO

HO

AAAD

Tyrosine

L-DOPA

Dopamine

NH2

CH2CHHO

HO

Dopamine

DflH

OH

Norepinehprine

DA

Exocytosis

DA Receptor

Reuptake

Action Potential

Vesicles

51

GO ON TO THE NEXT PAGE.

MCAT FL Test 11R 11/21/03 7:37 PM Page 51

138. Based on the information in the passage, which of thefollowing is a biologically active catecholamine?

A.

B.

C.

D.

139. Electrophilic aromatic substitution was performed ontyrosine and resulted in the following 2 products:

Which of the following best describes the rates offormation of these two products?

A. I forms faster than II.

B. II forms faster than I.

C. They have equal rates of formation.

D. The relative rates cannot be determined fromthis information.

H2N CH C

CH2

OH

O

OH

X

I II

H2N CH C

CH2

OH

O

OH

X

HO H

N

H H

HO

OH

H3C H

HO H

NH2

H H

HO

HO H

CH3

H H

OH

HO

HO H

CH3

H H

H2N

OH

52

GO ON TO THE NEXT PAGE.

MCAT FL Test 11R 11/21/03 7:37 PM Page 52

140. Which structure is the intermediate in the transfor-mation of dopamine to DOPAC?

A.

B.

C.

D.

141. MAO inhibitors are used as pharmaceutical treat-ments for depression. One effect of these drugs is thatthey cause an increase in the amount of dopaminepresent in the synaptic cleft. Which of the followingwould produce a similar response?

A. Inhibiting DβH.

B. Increasing the concentration of tyrosine.

C. Inhibiting the DA reuptake protein.

D. Increasing the concentration of AAAD.

142. The body uses norepinephrine (NE) both as a neuro-transmitter and as an endocrine hormone. Which ofthe following are true for norepinephrine?

I. NE is a steroid hormone

II. NE is associated with parasympathetic ner-vous system

III. The adrenal gland secretes NE

IV. NE contains a primary amino group

A. I and II

B. I, III, and IV

C. III and IV

D. II and IV

143. Which is the major product of free radical bromina-tion with dopamine?

A.

B.

C.

D.NH2

CH2

H2CHO

HO

Br

NH2

CH2

H2CHO

HO

Br

NH2

CH2HCHO

HO

Br

NH2

CH2

H2CHO

HO

Br

CH

OH

H2CHO

HO

CH

C

H2C

H

O

HO

CH

O

H2CHO

HO

CH3

H2CHO

HO

NH2

CH2

H2CHO

HO

Dopamine

Mono amine oxidase

Aldehyde dehydrogenase

CCH2 OH

O

HO

HO

DOPAC

?

53

GO ON TO THE NEXT PAGE.

MCAT FL Test 11R 11/21/03 7:37 PM Page 53

144. A student attempts to react 1,2-dihydroxybenzenewith 1-amino-2-chloroethane and AlCl3 in a Friedel-Crafts Alkylation to create dopamine.

This reaction fails because:

A. The amino group acts a base and removes a phe-nol proton, spoiling the reagent.

B. The carbocation complex created is prone torearrangements.

C. The hydroxyl groups are deactivators.

D. Both B and C.

NH2

CH2H2CHO

HO

Cl

AlCl3+

NH2

CH2

H2CHO

HO+ HCl

X

54

GO ON TO THE NEXT PAGE.

MCAT FL Test 11R 11/21/03 7:37 PM Page 54

Passage II (Questions 145–150)

The word progeria, derived from the Greek meaning“prematurely old”, is applied to two different genetic dis-eases which cause symptoms typically associated withaging. Hutchinson-Gilford syndrome, referred to as proge-ria of childhood, presents within the first two years of lifeand typically leads to a constellation of symptoms includ-ing but not limited to dwarfism, baldness, dry and wrinkledskin, stiff joints, hip dislocations, serious atherosclerosisand cardiovascular problems which often lead to deathbefore age 10. Werner syndrome is referred to as progeriaof the adult since the onset occurs after puberty. Patientswith Werner syndrome present with atherosclerosis andserious cardiovascular problems as well as cataracts,osteoporosis, diabetes, and malignancies. In spite of simi-larities in the appearance of affected individuals, each ofthese syndromes has distinct causes.

Hutchinson-Gilford syndrome results from a singledominant mutation of the lamin A gene on chromosome 1,and each case is believed to arise from a sporadic newmutation since parents and siblings are unaffected. Thisloss of the lamin A protein leads to irregular nucleusshapes and sizes as well as malformed nuclear membranes.The physical manifestations of Hutchinson-Gilfordbecome apparent early on with skin changes and a failureto gain weight in infancy. Because of the random nature ofthis mutation it is extremely rare but presents equally inboth sexes and all races.

Werner syndrome is also rare but is an autosomalrecessive disorder caused by a mutation in the WRN geneon chromosome 8 which leads to complete protein dys-function. The protein encoded by the WRN gene is a heli-case enzyme which functions specifically to unwind DNAfor replication. Dysfunctional helicase impairs replicationleading to an accumulation of DNA damage in cells, areduced replicative rate, symptoms of Werner syndromeand often death from cardiac problems or malignancies inthe mid 40s.

145. The high rate of malignancies seen in patients withWerner syndrome is most likely due to:

A. faulty DNA damage repair mechanisms.

B. a breakdown of the nuclear membrane.

C. damage caused to DNA during replication.

D. low levels of the WRN protein.

146. If the normal lamin A gene is represented as (+) andthe abnormal lamin A gene is represented as a (–),what is the genotype of the parents of a child affectedby Hutchinson-Gilford syndrome?

A. mother (+)(+), father (+)(+)

B. mother (–)(–), father (–)(–)

C. mother (+)(–), father (+)(–)

D. mother (+)(–), father (+)(+)

147. Hutchinson-Gilford syndrome is caused by the:

A. complete dysfunction of the lamin A protein.

B. overproduction of the lamin A protein.

C. underproduction of the lamin A protein.

D. normal production of the lamin A protein.

148. Which of the following would support the conclusionthat Hutchinson-Gilford is the result of a sporadicmutation?

A. The disease presents almost exclusively inmales.

B. The mutant gene is not present in either parentof an affected child.

C. The disease skips a generation in family pedi-grees.

D. The mutant gene is present in every generationin a family pedigree.

149. The mutation in the lamin A gene results in a proteinproduct that lacks 50 amino acids near the C termi-nus. Which of the following is a possible cause ofthis deletion?

A. A missense mutation

B. A back mutation

C. A sense mutation

D. A frameshift mutation

150. If a woman who is not a carrier of Werner syndromemarries a man who has the disease, what is thechance that their first daughter will be a carrier of thesyndrome?

A. 0%

B. 25%

C. 50%

D. 100%

55

GO ON TO THE NEXT PAGE.

MCAT FL Test 11R 11/21/03 7:37 PM Page 55

Passage III (Questions 151–157)

Humans have a closed circulatory system whichincludes a heart, arteries and veins; all structural compo-nents which contain blood. Endothelial cells provide com-munication between vessels and circulating blood and aregenerated from round endothelial progenitor cells (EPCs)which flatten out to form capillaries and line blood vessels.These important cells play an active role through the syn-thesis and secretion of substances such as nitric oxide andprostacyclin, which cause vasodilation, and endothelin,which causes vasoconstriction. EPCs are attracted to spe-cific areas by a protein called vascular endothelial growthfactor (VEGF), a type of platelet derived growth factorknown to be involved in the vascularization of the uterinewall in the menstrual cycle and in cancerous tumors byinducing angiogenesis, the process of new blood vesseldevelopment from existing vasculature.

The formation of blood vessels occurs at a rapid rate inan embryo. During embryonic development VEGF bindsto a receptor, VEGF-R2, causing multiplication and differ-entiation of EPCs into endothelial cells. Binding to VEGF-R1 then promotes the formation of the tubular vessel andangiopoietin factors allow the new vessel to recruit sur-rounding mesodermal cells, which will become the peri-cytes, and smooth muscles cells, which support the bloodvessel. A third receptor, neurophilin-1 which is known forguiding axonal growth within the nervous system, providesdirection to the growing blood vessel and steers it in thedirection of VEGF.

Mature endothelial cells continue to express thesereceptors. Cancerous tumors produce VEGF which dif-fuses to capillaries and binds the receptors causing newblood vessels to grow towards and into the tumor. Thistype of excessive angiogenesis is critical for the survivaland metastasis of cancer cells and also plays a major rolein diabetic blindness, rheumatoid arthritis, and otherpathological conditions.

151. In which of the following would VEGF not participate?

A. Wound healing

B. Platelet formation

C. Placental development

D. Tumor growth

152. Scientists engineered endothelial cells to containeither the VEGF-R1 and VEGF-R2 receptors or thetwo receptors plus the neurophilin-1 receptor.According to the information in the passage, whichof the following experimental results would beobserved if the endothelial cells were grown on a gra-dient of VEGF?

A. Cells with the neurophilin-1 receptor migratedmost rapidly up the gradient.

B. Cells without the neurophilin-1 receptormigrated most rapidly up the gradient.

C. Cells with the neurophilin-1 receptor migratedrandomly.

D. Cells with the neurophilin-1 receptor did notmigrate.

153. According to the passage, excessive angiogenesisplays a major role in diabetic blindness. Which of thefollowing is a mechanism by which angiogenesisleads to blindness?

A. New vessels sprout from mesodermal cells inthe eye covering the retina.

B. Vessels in the eye begin to close up and bloodflow to the retina is lost.

C. Existing capillaries sprout new branches thatinterfere with light transmission.

D. Capillaries in the eye break, leaking bloodwhich covers the retina.

154. Which of the following experimental results wouldbe observed if the VEGF-R1 receptor were chemi-cally blocked in an embryo?

A. EPCs will not differentiate into endothelialcells.

B. Mesodermal cells will not develop into smoothmuscle cells.

C. Axonal growth will not occur normally.

D. EPCs will not form a tubular structure.

155. Which of the following is a mechanism by whichtumors could produce VEGF?

A. Stimulating the production of nitric oxide.

B. Stimulating the production of angiopoietin.

C. Releasing cytokines which stimulate plateletproduction.

D. Releasing toxins to VEGF-R1.

56

GO ON TO THE NEXT PAGE.

MCAT FL Test 11R 11/21/03 7:37 PM Page 56

156. Damage to developing mesoderm in an embryo willlead to malformations in all of the following except:

A. blood

B. nervous system axons

C. arteries

D. pericytes

157. An inadequate supply of blood to the heart due toarterial blockage would most likely result in therelease of nitric oxide from endothelial cells inorder to:

A. protect endothelial cells.

B. increase blood pressure.

C. increase the flow of blood to the heart.

D. decrease the flow of blood to the heart.

158. Folic acid is absorbed in the proximal jejunum. All ofthe following are fat soluble vitamins that areabsorbed in the small intestine EXCEPT:

A. Vitamin D

B. Vitamin C

C. Vitamin K

D. Vitamin E

159. Which of the following has the lowest heat of hydro-genation per mole of H2 absorbed?

A. 1,3-heptadiene

B. 1,4-heptadiene

C. 1,2-heptadiene

D. 1,3,5,7-octatetraene

160. Glucagon release is stimulated by excess amino acidsin the blood. This leads to:

A. decreased ketone body synthesis

B. increased glycogen formation

C. increased gluconeogenesis

D. increased protein synthesis

161. Which of the following is NOT true regarding SN1and SN2 reactions?

A. SN1 reactions proceed via a carbocation inter-mediate; SN2 reactions are concerted.

B. SN1 reactions can yield rearrangement products.

C. The rates of SN1 reactions are largely affectedby steric factors; the rates of SN2 reactions arelargely affected by electronic factors.

D. SN1 reactions proceed more readily with a ter-tiary alkyl halide as reactant; SN2 reactions pro-ceed more readily with a primary alkyl halide asreactant.

162. The breakdown of body fat reserves in female ath-letes would involve which of the following metabolicpathways?

A. Glycolysis

B. Transamination

C. Beta oxidation

D. Lactic acid fermentation

Questions 158 through 162 are NOT basedon a descriptive passage.

57

GO ON TO THE NEXT PAGE.

MCAT FL Test 11R 11/21/03 7:37 PM Page 57

Passage IV (Questions 163–168)

Different flavors are identified by receptors on thetongue which are located on taste buds. Traditionally itwas held that there were four primary tastes, and thereforefour types of taste receptors, identified as sweet, sour, bit-ter, and salty. More complex tastes were only a combina-tion of these four with subtle effects mediated by olfaction.However, some scientists argue there is a fifth taste,umami, which is unique from the other four.

Umami, also known as monosodium glutamate(MSG), is the sodium salt of glutamic acid. Glutamateexists either bound in protein or as free glutamate such asMSG, which is present naturally in certain foods such asparmesan cheese and tomatoes. MSG can also be isolatedthrough the fermentation of corn or sugar cane and is usedas a food additive to enhance flavor. Its flavor is sometimesdescribed as ‘meaty’ or ‘savory’.

Glutamate is a nonessential amino acid which plays animportant role in many tissues and organs in the body. It isa neurotransmitter within the nervous system and there aremultiple receptors that bind it. One of these, mGluR4, isfound in taste buds.

One argument against umami as a fifth taste was theextreme sensitivity of mGluR4 to glutamate. However sci-entists have discovered that the mGluR4 is a truncatedversion of the glutamate receptor which is missing the ter-minal region which causes it to have decreased sensitivity.

T1R1+3 is another receptor found on the tongue whichworks in conjunction with taste mGluR4. This receptor isa heterodimer composed of the T1R1 and T1R3 proteins.This receptor recognizes a broad range of L-amino acidsand contributes to the perception of umami by enhancingits effect. The taste stimulus of umami is also enhanced inthe presence of nucleotides.

163. Which of the following must be true regarding thebrain mGluR4 receptor?

A. It is a truncated version of the glutamate receptor.

B. Its terminal region mediates strong binding ofglutamate.

C. Its binding of glutamate inhibits neuronalimpulses.

D. It works in conjunction with T1R1+3.

164. Which of the following would weaken the argumentthat umami is a unique receptor which constitutes afifth taste?

A. The receptor responds to glutamate at the sameconcentrations at which glutamate can be tasted.

B. Chemicals which mimic the taste of glutamateactivate the receptor.

C. The binding of glutamate hyperpolarizes tastereceptors.

D. The amount of glutamate in foods is muchgreater than the concentrations of glutamatefound in the brain.

165. Which of the following experimental results wouldbe expected if the taste mGluR4 receptor were thesame as the brain mGluR4 receptor?

A. Molecules which block the brain receptorinhibit the taste of umami.

B. Molecules which block the brain receptor do notinhibit the taste of umami.

C. Glutamate would only weakly bind the receptor.

D. The taste of umami would be milder than othertastes.

166. Aged cheese and ripened tomatoes contain moreumami than their fresh counterparts. This is due to:

A. Neural stimulation

B. T1R1+3 activation

C. Protein degradation

D. Taste mGluR4 activation

167. Based on information in the passage, which of thefollowing must be true?

A. The T1R1+3 receptor is specific for glutamate.

B. When present the umami taste overwhelms allother tastes.

C. Glutamate only binds a single receptor.

D. Glutamate is synthesized in synaptic terminals.

168. According to the passage the presence of nucleotidesenhance the effects of umami. From a biological per-spective what is the most likely reason for this?

A. To acquire nucleotides for DNA and RNAsynthesis.

B. To promote the dietary intake of amino acids.

C. To promote proper mineral uptake.

D. To enhance olfaction.

58

GO ON TO THE NEXT PAGE.

MCAT FL Test 11R 11/21/03 7:37 PM Page 58

Passage V (Questions 169–174)

Secondary messengers in biological systems are oftenthe targets of pharmacological treatments. The erectileresponse is one such system. Erectile dysfunction (ED)can be caused by a flaw in this messenger system. Via-gra™* (Sildenafil citrate) works to inhibit breakdown ofthe second messenger and thus helps create and maintainan erection.

An erection occurs when the arteries that carry bloodto the penis increase in diameter and blood flow to theorgan is increased. Nerves near these arteries signal themuscle of the vessel walls to relax, thus widening theopening.

When the Cavernous neurons receive the proper input,NO synthase converts L-Arginine to citrulline and nitricoxide in a calcium dependent process. NO gas, which istoxic if inhaled, acts as a neurotransmitter and diffuses intonearby cells where it complexes with the heme subunit ofSoluable Guanylate Cyclase. This enzyme converts amolecule of GTP to cyclic-GMP. cGMP activates kinasesthat relax the smooth muscle of the corpus cavernosum,thus creating an erection.

Phosphodiesterase 5 (PDE-5) breaks down cGMP toGMP thus inactivating the second messenger and stoppingthe erectile response. Two of the possible mechanisms forED involve this second messenger: (I) low levels of cGMPor (II) overactive breakdown of cGMP. Viagra™ (Silde-nafil citrate) works by competitively inhibiting PDE-5,thus increasing the concentration of cGMP.

Figure 1. Structure of Sildenafil

Figure 2

169. Viagra™ will not produce an erection in the absenceof sexual stimulation. Which of the following bestexplains this fact?

A. The PDE-5 enzyme is not expressed until thecavernous nerves are stimulated.

B. Viagra does not produce cGMP.

C. PDE-5 cannot be competitively inhibited underlow levels of NO.

D. Viagra increases production of NO.

H2N CH C

CH2

OH

O

CH2

CH2

NH

C

NH2

NH

H2N CH C

CH2

OH

O

CH2

CH2

NH

C

NH2

O

Nitric Oxide Synthase

NO

L-Arginine L-Citrulline

Fe (Heme)

Soluable Guanylate Cyclase

GTPcGMP + PPi

cGMP Kinase I

Relaxes smooth muscle of corpus cavernosum

Erection

PDE-5GMP

Sildenafil citrate

Inhibition

OEt

N

HNN

N

Me

nPr

O

O2S

N

N

Me

59

GO ON TO THE NEXT PAGE.

MCAT FL Test 11R 11/21/03 7:37 PM Page 59

170. If the NO producing neuron indicated below werestimulated, which of the other cells would beaffected?

A. II only

B. I, II, III

C. I only

D. II, III

171. Based on the passage, which of the following wouldbe a successful treatment for priapism (prolonged,painful erection)?

A. Administering a cGMP Kinase I agonist.

B. Injecting a NOS gene promoter.

C. Injecting intravenous somatostatin.

D. Administering phosphodiesterase-5.

172. Which of the following is a feasible explanation forthe toxicity of nitric oxide if inhaled?

A. NO binds to Hb, reducing its ability to carryoxygen.

B. NO alters the structure of surfactant and thusprevents gas exchange.

C. In the conditions present in the lungs, NO read-ily attacks O2 to give NO2, depleting the avail-able oxygen.

D. In the respiratory system, NO is a powerfulneuro-toxin.

173. Which of the following is likely to be coupled withNO release from the cavernous nerves?

I. Increase in heart rate

II. Release of epinephrine

III. Stimulation of the vagus nerve

A. I and II

B. I only

C. III only

D. I, II, and III

174. If the effects of NO occurred in arteries around thecirculatory system, all of the following would occurEXCEPT:

A. Cooling of the body.

B. Increase in blood pressure.

C. Decreased venous blood volume.

D. All of the above occur.

*Viagra is a trademark of Pfizer Pharmaceuticals

Axons

I

II

III

60

GO ON TO THE NEXT PAGE.

MCAT FL Test 11R 11/21/03 7:37 PM Page 60

175. Which of the following statements is true regardingthe relative boiling points of cis- and trans- 2,3-dibromobutene?

A. The boiling point of the cis isomer is higherbecause it has a net dipole moment.

B. The boiling point of the cis isomer is higherbecause it is thermodynamically more stable.

C. The boiling point of the trans isomer is higherbecause the π electrons are more delocalized.

D. The boiling point of the trans isomer is higherbecause it is more symmetrical.

176. All of the following are true of portal systemsEXCEPT:

A. The portal system may directly involve the pos-terior pituitary.

B. It involves 2 capillary beds connected by a vein.

C. They can exist both in the body and in the brain.

D. The portal system may directly involve thesmall intestine.

177. Stimulation of the sympathetic nervous systemwould most likely result in the release of:

A. nitric oxide

B. VEGF

C. prostacyclin

D. endothelin

178. Which of the following compounds would beexpected to have the highest boiling point?

A.

B.

C.

D.

179. The H—N—H bond angle in the quaternary ammo-nium ion is closest to:

A. 180°

B. 120°

C. 109°

D. 90°

CH3

CH3

N CH3

CH3

CH3

NH

CH3

CH3

CH3

CH3

CH3

Questions 175 through 179 are NOT basedon a descriptive passage.

61

GO ON TO THE NEXT PAGE.

MCAT FL Test 11R 11/21/03 7:37 PM Page 61

Passage VI (Questions 180–186)

Humans produce three types of endogenous brain opi-oids: enkephalins and β-endorphins, which are only pre-sent within the brain, and dynorphins, which are presentthroughout the central and peripheral nervous system.These three opioids are collectively known as endorphins.Endorphins are chemically very similar to exogenous opi-ates such as morphine and humans have three well-definedopiate receptors throughout the body, although there is evi-dence for subtypes of these receptors.

The mu receptor mediates the analgesic effects of opi-ates at the level of the thalamus, but mu receptors are alsofound in the respiratory centers of the brain and in largenumbers in the GI tract. Opiates depress respiratory rateand depth as well as intestinal peristalsis. They also inhibitthe impulse to cough. The mu receptor mediates the feel-ing of euphoria which is a major contributor to the addic-tiveness of opiate drugs.

Psychotomimetic effects, such as hallucinations, aremediated by the delta receptor. The kappa receptor medi-ates sedation and analgesia at the spinal level as well asmiosis of the pupil. However both delta and kappa recep-tors are present in the vomiting center of the brain. Stimu-lation by opiates leads to nausea and vomiting althoughwith regular use tolerance develops as the normal responseto a drug is reduced.

Drugs that act on opiate receptors can either be a nar-cotic agonist, a mixed agonist-antagonist, or an antagonist.These drugs can be natural opiates, semisynthetic analogs,or synthetic compounds and have similar molecular struc-tures as endogenous opiates. The mixed agonist-antagonistdrugs have an agonist effect one some opiate receptors andan antagonist effect on others and their actions at a certaintype of receptor can vary with its location in the body. Thisclass of drug also includes partial agonists.

180. Which of the following could not be treated throughstimulation of an opiate receptor?

A. Constipation

B. A severe cough

C. Diarrhea

D. Pain

181. Which of the following actions of a mixed agonist-antagonist drug would be most useful to create anal-gesia without vomiting?

A. Mu: agonist; Delta: agonist; Kappa: antagonist

B. Mu: antagonist; Delta: antagonist; Kappa: agonist

C. Mu: agonist; Delta: antagonist; Kappa: antagonist

D. Mu: agonist; Delta: antagonist; Kappa: agonist

182. Morphine is an exogenous ligand of the mu receptorand it also releases histamine. Which of the followingwould not be an effect of morphine use?

A. Red skin

B. Euphoric feelings

C. Vomiting

D. Constipation

183. Naloxone is an opiate receptor antagonist which canbe used to treat an opiate overdose or an opiate addic-tion. What is the method of action of Naloxone?

A. Naloxone binds the opiate drug blocking itsbinding to the receptor.

B. Naloxone binds opiate receptors blocking thebinding of opiate drugs.

C. Naloxone inhibits the effects of opiates afterthey have bound receptors.

D. Naloxone inhibits the release of endogenousopiates.

184. Mixed agonist-antagonist drugs are being developedin the hopes of utilizing the full anesthetic and seda-tive properties of opiates while avoiding opiateaddiction. Which of the following drug actions wouldachieve this?

A. Antagonist at mu receptors; agonist at kappareceptors

B. Partial agonist at delta receptors; agonist at mureceptors

C. Agonist at mu receptors; antagonist at deltareceptors

D. Partial agonist at mu receptors; agonist at kappareceptors

62

GO ON TO THE NEXT PAGE.

MCAT FL Test 11R 11/21/03 7:37 PM Page 62

185. Which of the following actions would induce spinalanalgesia?

A. Enkephalins at mu receptors

B. β-endorphins at delta receptors

C. Dynorphins at kappa receptors

D. Enkephalins at kappa receptors

186. The drug morphine stimulates enkephalin receptorsintensely. This stimulation by an exogenous opiatepromotes a negative feedback mechanism whichinhibits the production of endogenous enkephalin.This will lead to:

A. an inhibition of morphine’s effects.

B. an amplification of morphine’s effects.

C. tolerance to morphine.

D. no effect.

63

GO ON TO THE NEXT PAGE.

MCAT FL Test 11R 11/21/03 7:37 PM Page 63

Passage VII (Questions 187–192)

Aspartame is a highly controversial artificial sweet-ener which has generated enormous concern over itsprevalent usage in many foods and beverages. There is awealth of anecdotal evidence which blames aspartame foreverything from infertility to brain tumors. To understandthe hysteria, the chemical composition of aspartame mustbe examined.

Aspartame is 180 times sweeter than sucrose. In orderto achieve a certain level of sweetness the amount of aspar-tame needed is less than 1% the amount of sugar needed.Aspartame is a dipeptide, composed of two amino acids(phenylalanine and aspartic acid) and methanol; whenmetabolized within the body it is broken down into thesethree components.

Aspartame is 40% aspartic acid by weight and cancause brain damage in large doses since it acts like an exci-tatory neurotransmitter in the brain. However, in normaldoses aspartic acid is prevented from entering the brain bythe blood-brain barrier.

Methanol comprises 10% of aspartame by weight andis further metabolized in the liver by alcohol dehydroge-nase to formaldehyde which can diffuse into tissues andform cross links between amino acids abrogating their nor-mal function. However, formaldehyde does not remain inthe body long as it is metabolized to formic acid whichinhibits the cytochrome oxidase complex of mitochondriaand is consequently very toxic to humans.

Methanol and proteins such as aspartic acid are presentin small quantities in other foods. However, the questionregarding the safety of aspartame relies on the amountingested. Studies have shown that the amount of aspartameneeded to achieve toxicity is far beyond normal humanconsumption, and most likely not even possible for ahuman to consume in the necessary time frame. However,individuals with the rare genetic disorder Phenylketonuria(PKU) should not consume aspartame due to its pheny-lalanine content.

187. Which of the following is true regarding themetabolic breakdown products of methanol?

A. Formaldehyde is dangerous but formic acid isharmless.

B. Methanol metabolism takes place in the smallintestine.

C. Formaldehyde and formic acid are more danger-ous than methanol.

D. Formic acid interferes with normal protein func-tioning.

188. The passage states that formic acid disrupts the func-tion of mitochondria which is toxic to humans. Thisdisruption of the mitochondria causes:

A. excessive energy production which leads to celldeath.

B. the rupture of cells due to the buildup of energyprecursors.

C. cell death by directly interfering with the citricacid cycle.

D. lack of cellular energy due to decreased glyco-gen degradation.

189. Aspartic acid acts as an excitatory neurotransmitterin the brain but causes neuronal cell death if it is pre-sent in large quantities. This death is caused by:

A. the breakdown of aspartic acid

B. excessive calcium influx

C. the presence of excess glutamate

D. inhibition of neural impulses

190. A common result of methanol poisoning is blind-ness. Which of the following can be inferred fromthis information?

A. The visual center of the brain is very sensitive toformaldehyde.

B. Retinal vessels are very sensitive to formalde-hyde.

C. Vitreous humor is very sensitive to formic acid.

D. The optic nerve is very sensitive to edema.

64

GO ON TO THE NEXT PAGE.

MCAT FL Test 11R 11/21/03 7:37 PM Page 64

191. Phenylalanine is an essential amino acid that ismetabolized to tyrosine by phenylalanine hydroxy-lase. Tyrosine ultimately forms fumarate, which isused in the citric acid cycle, and acetoacetate. PKU isa rare disorder which results from a deficiency ofphenylalanine hydroxylase and results in alternatephenylalanine products such as phenyllactate andphenylacetate. Which of the following would be ele-vated if a PKU patient ingested aspartame?

A. Phenylalanine hydroxylase and tyrosine

B. Fumarate and acetoacetate

C. Phenyllactate and phenylalanine

D. Phenylalanine and phenylalanine hydroxylase

192. Which of the following would be expected inmethanol poisoning?

A. Metabolic acidosis

B. Metabolic alkalosis

C. Hypoventilation

D. Decreased anion gap

65

GO ON TO THE NEXT PAGE.

MCAT FL Test 11R 11/21/03 7:37 PM Page 65

Passage VIII (Questions 193–199)

The Primary Somatosensory (SI) cortex encompassesthe areas of the cerebrum which are responsible for inter-preting tactile stimuli such as touch, pain, pressure, andtemperature. More area in the SI cortex is dedicated toregions of the body with the highest innervation densitiessuch as the feet, hands, face, and lips. A rough map of theSI cortex places body parts next to each other in the fol-lowing order: internal organs, head and face, hand, upperarm, upper torso, lower trunk, lower leg, foot, genitals.

Often individuals who have lost a limb through ampu-tation complain of sensations in the missing limb, particu-larly in areas which have the greatest representation in theSI cortex. This is known as Phantom Limb Syndrome andits presentation varies widely from benign feelings of thelimb still being present to persistent and debilitating painin the missing limb. Although neurons within the peripheryand spinal cord may modulate this syndrome to a smalldegree the cause of the phantom limb is in the brain.

After amputation of a limb the corresponding region inthe SI cortex experiences a massive disruption of sensoryinput. This dramatic disruption causes the brain to attempta reorganization of the SI cortex and it appears that normalareas of the cortex make contact with areas that are nowdeprived of sensations. Contact usually comes betweenareas that are closely associated in the cortex and this canlead to feelings of stimulation in the missing limb whenother areas of the body are touched; for example a personmissing an arm may feel as though the amputated limb isbeing touched when their face is touched.

193. The theory that Phantom Limb syndrome is causedby reorganization of the SI cortex relies on theassumption that:

A. the brain is highly unorganized.

B. the neurons in the cortex can grow and makenew connections.

C. neurons not in use will atrophy within the brain.

D. neurons are unable to make new connections.

194. Which of the following is true regarding PhantomLimb syndrome which results in chronic pain in thephantom limb?

A. The pain results from a physical cause.

B. The pain is not real; it is only imagined by thepatient.

C. The pain results from a malfunctioning nervoussystem.

D. The pain causes a reorganization of the SI cor-tex.

195. Based on information and the example in the pas-sage, if a tracer compound were injected into the chinof an amputee who had lost an arm which of the fol-lowing results would most strengthen the reorganiza-tion theory?

A. The tracer appears only in the brain region asso-ciated with the chin.

B. The tracer appears only in the brain regionsassociated with the hand and arm.

C. The tracer appears in brain regions associatedwith the chin, hand and arm.

D. The tracer appears in brains regions associatedwith the chin, trunk and hand.

196. Which of the following experimental observationswould suggest that Phantom Limb syndrome is notcaused by reorganization of the SI cortex as sug-gested in the passage but instead results from thespinal cord?

A. The affected area of the cortex shows regrowth.

B. The affected nerves in the spinal cord showspontaneous electrical activity.

C. The affected area of the cortex responds whenadjacent areas are stimulated.

D. The affected nerves in the spinal cord are inac-tive.

66

GO ON TO THE NEXT PAGE.

MCAT FL Test 11R 11/21/03 7:37 PM Page 66

197. The pathway that transmits pain runs from theperiphery through the thalamus where it is then pro-jected to the appropriate cortical region. The thala-mus has a high density of opiate receptors while thecortex has a low density. Which of the followingobservations would strengthen the argument thatphantom pain is a result of cortical reorganization?

A. Phantom pain results after manipulation of theproximal stump.

B. Phantom pain begins immediately after amputa-tion.

C. Phantom pain is relieved through the use ofanalgesic drugs.

D. Phantom pain is not relieved through the use ofanalgesic drugs.

198. According to the passage the most likely explanationfor the observation in the third paragraph that anamputee whose face is touched feels as though theirarm is touched is:

A. nerves in the stump are stimulated when the faceis touched.

B. the arm area of the cortex has atrophied.

C. neurons in the facial area of the cortex stimulatethe arm area.

D. nerves in the face have grown to connect withthe arm’s nervous pathways.

199. Which of the following could also possibly lead toPhantom Limb syndrome?

A. Amputation of a finger or toe

B. Paralysis of an arm or leg

C. Damage to the SI cortex

D. Breaking an arm or leg

200. In the electrophilic aromatic substitution of phenol,substituents add predominantly in which position(s)?

A. A

B. B

C. C

D. A and B

201.

A. SN1/E1

B. SN2/E2

C. SN1

D. SN2

CH3

H3 H2C

Cl

H

+ NaOH ?

C

N(CH3)2

A

C

B

Questions 200 through 204 are NOT basedon a descriptive passage.

67

GO ON TO THE NEXT PAGE.

MCAT FL Test 11R 11/21/03 7:37 PM Page 67

202. If a neuron were isolated from the brain and trans-ferred to similar, more concentrated, environment,how would the sodium-potassium pump respond?

A. Its rate would increase

B. Its rate would decrease

C. Its rate would not change

D. It would not be involved

203. All of the following result in the production ofdiploid cells EXCEPT:

A. Fertilization

B. Mitosis

C. Parthenogenesis

D. Budding

204. Which of the following is a correct association repre-sentative of intracellular organelles?

A. Intermediate filaments: form centrioles

B. Centrioles: located in the centrosome area ofplant cells

C. Glyoxysomes: use oxidative enzymes to pro-duce hydrogen peroxide

D. Smooth Endoplasmic reticulum: Detoxify drugs

68

GO ON TO THE NEXT PAGE.

MCAT FL Test 11R 11/21/03 7:37 PM Page 68

Passage IX (Questions 205–209)

Carbohydrates are comprised of polyhydroxy aldehydeor ketone units called monosaccharides of the formulaCx(H2O)y. The most common monosaccharides arealdopentoses, ketopentoses, aldohexoses and ketohexoses.Monosaccharides are, by convention, written using Fischerprojections to denote the linear form and drawn in theHaworth form to depict the cyclic form.

In the late 1800s Emil Fischer began deducing thestereochemical structure of (+)-glucose (now known as D-(+)-glucose). At that time Fisher knew that (+)-glucosewas an aldohexose. He was also aware of a few more facts:the compounds (+)-mannose (an aldohexose) and (+)-ara-binose (an aldopentose) existed and that aldohexoses havefour stereocenters with a total of 16 possible stereoiso-mers. Fisher surmised (+)-glucose was one of these 16stereoisomers. He then decided to limit his possibilities toonly the structures with the D-configuration, those with thehydroxy group of the penultimate carbon drawn to theright (listed below).

Fisher also knew that the structures of glyceraldehydeand arabinose had three and four carbons, respectively.Fisher experimentally determined that the nitric acid oxi-dation of both ends to carboxylic acids of (+)-glucose withnitric acid gave an optically active aldaric acid; (+)-man-nose also yielded a different optically active aldaric acid.

Using the Kiliani-Fischer Synthesis reaction, whichlengthens the chain of an aldose at the aldehyde group,

Fischer determined that (–)-arabinose formed a mixture of(+)-glucose and (+)-mannose. Fischer also developed amethod of chemically interchanging the two end groups(–CHO and –CH2OH). After interchanging both ends of(+)-glucose he found that (+)-glucose gave L-gulose and(+)-mannose remained (+)-mannose.

205. Nitric acid oxidizes the aldehyde (CHO) and primaryalcohols (CH2OH) of sugars to carboxylic acids(COOH) or aldaric acids. Which compounds (1–8)would NOT be optically active aldaric acids afteroxidation by nitric acid?

A. Compounds 1 and 7

B. Compounds 2 and 8

C. Compounds 3 and 5

D. Compounds 4 and 6

206. The Kiliani-Fischer Synthesis reaction lengthens thechain of an aldose at the aldehyde group. Which struc-tures could Fischer determine were synthesized from(–)-arabinose using the Kilaini-Fischer Synthesis?

A. Compounds 1 and 2

B. Compounds 3 and 4

C. Compounds 5 and 6

D. Compounds 7 and 8

207. Fischer determined that chemically interchanging thetwo end groups of (+)-mannose yielded (+)-mannose.Which compound(s) could be a possible structure for(+)-mannose?

A. Compound 1 only

B. Compound 4 only

C. Compound 4 and 6

D. Compounds 4 and 7

CHO

HHO

OHH

OHH

CH2OH

CHO

H OH

HO H

HO H

CH2OH

OHH

CH2OH

D-(-)-arabinose

CHO

L-(+)-arabinoseD-(+)-glyceraldehyde

CHO

HO H

H OH

HO H

HO H

CH2OH

L-gulose

CHO

OHH

OHH

OHH

OHH

CH2OH

CHO

HHO

OHH

OHH

OHH

CH2OH

CHO

OHH

HHO

OHH

OHH

CH2OH

CHO

HHO

HHO

OHH

OHH

CH2OH

(1) (2) (3) (4)

CHO

OHH

OHH

HHO

OHH

CH2OH

CHO

HHO

OHH

HHO

OHH

CH2OH

CHO

OHH

HHO

HHO

OHH

CH2OH

CHO

HHO

HHO

HHO

OHH

CH2OH

(5) (6) (7) (8)

69

GO ON TO THE NEXT PAGE.

MCAT FL Test 11R 11/21/03 7:37 PM Page 69

208. Aldoses and ketoses can be reduced with NaBH4 toform compounds called additols. Additols have pri-mary alcohol groups at both ends. Which com-pound(s) (1–8) would yield optically INACTIVEadditols?

A. 1 only

B. 1 and 4 only

C. 4 and 6 only

D. 1 and 7 only

209. Osazone formation using phenylhydrazine results ina loss of stereocenter at C2 for aldohexoses. Deter-mination of epimeric compounds at C2 is possiblebecause osazone formation leaves the C3, C4, C5,and C6 hydroxyls of an aldohexose unchanged. Thereaction also occurs with ketoses. The ketohexose D-sorbose would appear to be epimeric with whichaldohexose(s) using phenylhydrazine(1–8).

A. 5 only

B. 6 only

C. 5 and 6

D. 7 and 8

CH2OH

H OH

HO H

H OH

O

CH2OH

D-sorbose

70

GO ON TO THE NEXT PAGE.

MCAT FL Test 11R 11/21/03 7:37 PM Page 70

Passage X (Questions 210–214)

Primary amines can be synthesized by treating alkylhalides with ammonia followed by an abstraction of protonfrom the quaternary ammonium salt. The reaction rate isdependent on the concentrations of both ammonia and thealkyl halide.

NH3 + CH3CH2—Cl ➝ CH3CH2—NH3+Cl–

Reaction 1

CH3CH2—NH3+ Cl– + NaOH ➝ CH3CH2—NH2 + NaCl

+H2O

Reaction 2

This method, however, is no synthetically efficientbecause multiple alkylations can occur. The ethylaminiumchloride that is produced in reaction 1, for example, canreact with ammonia that is present in the solution to produceethylamine. Ethylamine can then compete with ammoniain reacting with ethyl chloride eventually producing a sec-ondary amine. Tertiary amines as well as quaternaryammonium salts can also from via a similar mechanism.

An alternative method, termed reductive amination ofthe aldehyde or ketone, can be used to selectively synthe-size primary, secondary or tertiary amines. Using thismethod one can prepare ethylamine by treating acetalde-hyde with ammonia followed by a treatment with a reduc-ing agent such as NaBH3CN or LiBH3CN (sodium orlithium cyanoborohydride).

Reaction 3

210. Which of the following products is not expected toform in the reaction of NH3 with propyl bromide?

A. 1-Aminopropane

B. N-Propyl-2-aminopropane

C. N-Propyl-1-aminopropane

D. N,N-Dipropyl-1-aminopropane

211. Which of the following represents an increasingorder of basicity of amines in an aprotic solvent?

A. (CH3) 3N, (CH3) 2NH, CH3NH2, (CH2Cl) 2NH

B. (CH2Cl) 2NH, (CH3) 2NH, CH3NH2, (CH3) 3N

C. (CH2Cl) 2NH, CH3NH2, (CH3) 2NH, (CH3) 3N

D. (CH3) 3N, CH3NH2, (CH3) 2NH, (CH2Cl) 2NH

212. The formation of ethylaminium chloride from ethylchloride and ammonia proceeds by:

A. an SN1-type nucleophilic substitution.

B. an SN2-type nucleophilic substitution.

C. an E1-type elimination.

D. an E2-type elimination.

213. Ammonia can react readily with carboxylic acids andcarboxylic acid derivatives forming various substitutionproducts. What product would you expect to be formedin the reaction of ammonia with acetyl chloride?

A. Acetamide

B. Acetylamine

C. Acetic acid

D. Chloroacetamide

214. Which of the following is most probably the interme-diate formed during the reaction shown in reaction 3.

A.

B.

C.

D.

STOP. IF YOU FINISH BEFORE TIME IS CALLED,CHECK YOUR WORK.YOU MAY GO BACK TO ANYQUESTION IN THIS SECTION ONLY.

CH3

CH3 NH2

OH2+

CH3 NH2

O

CH3

H NH3+

OH

CH3

CH3 NH3+

OH

CH3 H

ONH3

NaBH3CN or LiBH3CNCH3 NH2

71

MCAT FL Test 11R 11/21/03 8:17 PM Page 71

MCAT FL Test 11R 11/21/03 7:51 PM Page 73

MCAT FL Test 11R 11/21/03 7:37 PM Page 72

I.N. MM3264A – Printed in U.S.A.

Full Length Test 11R 11/21/03 5:11 PM Page 2


Recommended